You are on page 1of 52

~~-----------------------

1. A projectile can have the same range R for two angles of projection. If t 1 and t 2 be the times of flights in the two cases, then the product of the two times of flights is proportional to ;

(a) R 2 (b) _!_z

. R

(e) _!_ Cd) R

R

2. An annular ring with inner and outer radii Rj and R2 is rolling without slipping with a uniform angular speed. The ratio o-f the forces experienced by the two particles situated on

the inner and outer parts of the ring, ;~ is :

(e) 1

3. A smooth block is released at rest 'on a 450 incline and then slides a distance d. The time taken to slide is n times as much to slide on rough incline than on a smooth incline. The coefficient of friction is : .

(a) Ilk'" 1- _!_ (b) ""k = ~1 _ 1

n2 n2

(c) 11 s '" 1 - ~ (d) .. 1-l J =:: ~1 - \

n '. n

4. The upper half of an inclined plane with inclination $ is perfectly smooth, while the lower half is rough. A body starting from rest at the top wil1 again come to rest at the bottom, if the coefficient of friction for the lower half is gi~en by: (a) 2sin ijl (e) 2 tan ~

(b) 2 cos ~ (d) tan ~

5. A bullet fired into a fixed target loses half of its velocity after penetrating 3cm. How much further it will penetrate before coming to rest, assuming that it faces constant resistance to motion?

(a) 3.0 em (c) 1.5 em

'Cb)_2.Q_cm Cd) 1.0 em

6. Out of the following pairs, which one does not have identical dimensions?

(a) Angular momentum and Planck's constant (b) Impulse and momentum

(e) Moment of inertia and moment of a force (d) Work and torque

7. The relation between time t and distance x is t ee a.'{'z + bx, where a and b are constants. The

acceleration Is : (a) - 2aby2

(c) - 2av3

(b) 2bv3 (d) 2aJl2

8. A car, starting from rest, accelerates at the rate I through a distance S, then continues at constant speed for time t and then decelerates at the rate 1/2 ro come to rest. If the total distance travelled is 155, then:

1 . 2 (b) S='6lt

Cd) S=.! It2 4

(a) 5 -= Ie

1 I 2 (c) 5 == - t

2

9. A particle is moving eastwards with a velocity . of 5 O1s-J• In 10 s the velocity changes to 5 ms! northwards. The average acceleration in this time is :

Ca) 1 ms-2 towards north-east

1 - -

(b) "2 rns-2 towards north

(c) zero

1 .

(d) '2 ms-2 towards north-west

10. A parachutist after bailing out falls 50 m without friction. When parachute opens, it decelerates at 2 m/ 52. He reaches the ground with a speed of 3 m/s, At what height, did he baH out?

(a) 91 m (b) 182 m

(c) 293 m . (d) 111 m

11.

A block is kept on a frictionless inclined surface with angle of inclination n, The incline is given an acceleration a to keep the block stationary. Then a isequal to :

(a) gltan IX (b) g cosec a

(c) _g Cd) s tan a

12. A sphericalball of mass 20 kg is stationary at the top of a hill of height 100 m, It rolls down a smooth surface to the ground, then climbs up another hill of height 30 m and finally rolls down to a horizontal base at a height of 20 rn above the ground. The velocity attained by the ball is :.

(a) 40 m/s (b) 20 m/s

(c) 10 rn/s Cd) wJ36" rn/s

13. A body A of mass M while falling vertically downwards under gravity breaks into two

parts; a body B of mass .~ M and, a body C of

mass j M. The. centre of mass of bodies B. and C taken together shifts compared to that of body A towards:

(a) depends on height of breaking (b) does not shift

(c) bodyC

(d) body B

14. The moment of inertia of uniform. semicircular disc of mass M and radius r about a line perpendicular to the plane of the disc through the centre is:

(a) ~Mr (c) M,-l

Cb) ~M? 5

(d) ~ M,-l

15. A particle of mass 0.3 kg is subjected to a force F = - kx with k = 15 N/m. What will be its initial acceleration, if it is released from a point 20 em away from the origin?

(a) 3m/S2 (b) ISm/s2

(c) 5 m/s2 Cd) 10 m/s2

16. . The block of mass M moving on the frictionless horizontal surface collides with the spring of

spring constant k and compresses it by length L. The maximum momentum of the block after collision is :

M J

711~/I/r:99999~ .

(a) JMk L

kL2 (b) 2M

Cd) Me k

(cj zero

A mass m moves with a velocity v and collides inelastically with another identical mass. After

collision the I" mass moves with velocity ~. in

. ,,3 .

a direction perpendicular to the initial direction of motion. Find the speed of the second mass after collision :

Ca) v (b) 13 v

(c) Jj v Cd) J-J

18. A 20 em long capillary tube is dipped in water, The water rises upto 8 ern, If the entire arrangement is put in a freely faIHng elevator, the length of water column in the capillary tube will be:

(a) 8 em (b) 10 em

(c) 4 em (d) 20 em

1 '7.

19. If S is stress and Y is Young's modulus of material ofa wire, the energy stored in the wire per unit volume is :

S2 (b) 2Y

S (d)2Y

20. Average density of the earth: (a) does not depend on g

(b) is a complex function of g (c) is directly proportional to g Cd) is inversely proportional to g

21.. A body of mass m is accelerated uniformly from rest to a speed v in a time T. The instantaneous power delivered to the body as a function of time, is given by:

mv2 mv2

(a) --2 t (b) __ (2

T T2

(c) .! mv~ t Cd) .!2· m~2 t 2

2 T2 . T

22. Consider a car moving on a straight road with a speed of 100 m/s, The distance at which car can be stopped, is : [~l k '" 0.5)

(a) 800 m (b) 1000 m

(c) 100 m (d) 400 m

23. Which of the following is incorrect regarding the first law of thermodynamics?

(a) It is not applicable to any cyclic process (b) It is a restatement of the principle of

conservation of energy

(c) It introduces the concept of the internal energy

(d) It introduces the concept of the entropy

24. A T shaped object with dimensions shown i!J the figure, is lying on a smooth floor. A force F is applied at the point P parallel to AB, such that the object has only the translational motion without rotation. Find the location of P with respect to C :

~ F

tJ
p 1
1
c
(b) ~l
(d) I 2 (a) "31

4 (c) "31

25. The change in the value of g at a height h above the surface of the earth is the same as at a depth d below the surface of earth. When both d and h are much smaller than the radius of earth, then which one of the following is correct ?

Ca) d =!!:. (b) d '" 3h

2 2

(c) d = 2h

Cd) d = It

,26. A particle of mass 10 g is kept on the surface of a uniform sphere of mass 100 kg and radius 10 em, Find the work to be done against the gravitational force between them, to take the particle far away from the sphere : (you may take G '"' 6.67 xlO-ll Nm2/kg2)

(a) 13.34x 10-10 J (b) 3.33)( 10-10 J

(c) 6.67 x 10-9 J Cd) 6.67 x 10-10 J

A gaseous mixture consists of 16 g of helium and C

16 g of oxygen. The ratio C P of the mixture is :

v

(b) 1.62 (d) 1.54

27.

(a) 1.59 (c) 1.4

The intensity of gamma radiation from a given source is I. On passing through 36 mm of lead, it is reduced to 1/8. The thickness of lead, which will reduce the intensity to 112 will be :

(a) 6 mm (b) 9 mm

(c) 18 mm Cd) 12 rnm

The electrical conductivity of a semiconductor increases when electromagnetic radiation of wavelength shorter than 2480 nm, is incident on it. The band gap in (eV) for the semiconductor is:

(a) 1.1 eV (b) 2.5 eV

(c) 0.5 eV Cd) 0.7 eV

A Photocell is illuminated by a small bright source placed 1 m away. When the same source

of light is placed ~ m away, the number of

electrons emitted by photocathode would:

(a) decrease by a factor of 4

(b) increase by a factor of 4

(c) decrease by a factor of 2

(d) increase by a factor of 2

Starting with a sample of pure 66Cu, 7/8 of it decays into Zn in IS min. The corresponding half-life is :

(a) 10 min (c) 5 min

28.

29.

30,

31.

(b) 15 min (d) 7 ~ min

If radius of the N Al nucleus is estimated to be 3.6 fermi, then the radius of 1~sTe nucleus be nearly:

(a) 6 fermi (c) 4 fermi

32.

(b) 8 fermi Cd) 5 fermi

The temperature-entropy diagram of a reversible engine cycle is given in the figure. Its efficiency is :

T

33.

(a) 1/2 (c) 1/3

s

o» 1/4 (d) 2/3

34. The figure shows a system of rwo concentric spheres of radii r, and rz and kept at ternperatures Tj andTz> respectively. The radial rate of now of heat in a substance between the two concentric spheres, is proportional to :

(b) In (~)

(d) (r2 -Ij)

35.

A system goes from A to- B via two processes I and II as shown in figure. If AUt and 6U2 are the changes in internal energies in the processes I and II respectively, then:

p

II A<:::=>S I

v

(a) 6.UI = 6U2

(b) relation between 6UI and 6Uzcannoe be determined

(c) I1U2 > 6UI

(d) 6U2 < 6UI

3&. The function sin2 (oot) represents :

(a) a periodic, but not simple harmonic, motion with a period 211:/(0

(b) a periodic, but" not simple harmonic, motion with a period 11:/00

(c) a simple harmonic motion with a period 2rr/ro

(d) a simple harmonic motion with a period 'It/ro

37. A Young's double slit experiment uses a monochromatic source. The shape of the interference fringes formed on a screen is :

(a) hyperbola (b) circle

(c) straight line (d) parabola

38. Two simple harmonic motions are represented by the equations y, '" 0.1 sin (100 1t t + %) and

Y2 '" 0.1 cos -cr. The phase difference of the velocity of particle 1, with respect to the

velocity of panicle .2 is : .

-Jt x

(a) 6" (b) "3

(e) - 'It (d) .::.

3 6

39. A fish looking up through the water sees the outside world, contained in a circular horizon.

If the refractive index of water is ~ and the fish is 12 em below the water surface, the radius of this circle in em is:

(a) 36,{7 (e) 36.[5

(b) 36 ..fi

(d) 4.[5

40.

Two point white dots are 1 mm apart on a black paper, They are viewed by eye of pupil diameter 3 mm. Approximately, what is the maximum distance at which these dots can be resolved by the eye? [Take wavelength of light '" 500 nm]

(a) 5 m (b) 1 m

(e) 6 m Cd) 3 m

A thin glass (refractive index 1.5) lens has optical power of - 5 D in air. Its optical power in a liquid medium with refractive index 1.6 will be:

(a) I D (b) - 1 0

(c) 250 Cd) - 250

The diagram shows the energy levels for an electron in a certain atom. Which transition shown represents the emission of a photon with the most energy ?

41.

42.

~-~~------,.----- n '" 4

T n=3

__,_~-~-_,,_~-- a= I

(a) III ee) I

II III IV

(b) IV Cd) II

43. If the kinetic energy of a free electron doubles, its de-Broglie wavelength changes by the factor:

(a) ~ (b) 2

l' r:

(c) r::. (d) ..;2

-..;2 .

44. In a common base amplifier, the phase difference between the input signal voltage and output voltage is :

(a) ~ (b) rr

4

(c) zero

(d)1r 2

45. In a full wave rectifier circuit operating from 50 Hz mains frequency, the fundamental frequency in the ripple would be :

(a) 50 Hz (b) 25 Hz

(c) 100 Hz (d) 70.7 Hz

46. A nuclear transformation is denoted by X(n, a)-i> jLi. Which of the following is the

nucleus of element X?

(a) ~2C (b) 1°B

(c) ~B (d) ~IBe

47. A moving coil galvanometer has 150 equal divisions. Its current sensitivity is 10 divisions per milliampere and voltage sensitivity is 2 divisions per millivolt. In order that each division reads 1 V, the resistance in Ohm's needed to be connected in series with the coil will be :

(a) 103 (b) 105

(c) 99995 (d) 9995

48. Two voltameters, one of copper and another of silver, are joined in parallel. When a total charge q flows through the voltameters, equal amount of metals are deposited. If the electrochemical equivalents of copper and silver are zi and 22 respectively, the charge which flows through the silver voltameter is :

(a) _q_ (b) _q_

1 + ~ 1 + Z2

22 21

ZJ d 22

(c) q_ ( ) q~zl

Z2

49. In the circuit, the galvanometer G shows zero deflection. If the batteries A and B have negligible internal resistance, the value of the resistor R will be :

500n

R

A

(a) 200n (c) soon

(b) lOon (d) 1000n

50. Two sources of equal emf are connected to an external resistance R. The internal resistances of the two sources are Rl and R2 (R2 >Rj). If the potential difference across the source having internal resistance R~ is zero, then:

(a) R '" Rz X (R1 +. 2) (R2 - RI)

(0) R o=R2 - Rl

ec) R R]Rz

CRI + R2)

Cd) R "" R]RZ

(R2 - Rj)

51. A fully charged capacitor has a capacitanceC.lt is discharged.through a small coil of resistance wire embedded in a thermally insulated block of specific heat capacity s and mass m. If the temperature of the block is raised by I1T, the potential difference V across the capacitance is :

(a) ~2m~I1T (b) m~6T

(c) IlliCAT (d) ~2rn::AT

52. One conducting U -tube can slide inside another as shown in figure, maintaining electrical contacts between the tubes. The magnetic field B is perpendicular to the plane of the figure. If each tube moves towards the other at a constant speed v, then the emf induced in the circuit in terms of B, 1 and v, where I is [he width of each tube, will be :

x B

x ~A~ ~~ ~

x

x

x

x

x D

c

(a) Blv (c) zero

(b) -Blv (d) 2Blv

53. A heater coil is cut into two equal parts and only one pan is now used in the heater. The heat generated will now be ;

(a) doubled (b) four times

(c) one-fourth (d) halved

54. Two thin, long, parallel wires, separated by a distance d carry a current of i A in. the same direction. They will :

·2 (a) attract each other with a force of rz~d)

·2 (b) repel each other with a force of rz~)

·2

'(c) attract each other with a force of Jl rJ. 2. '

(2nd )

·2

(d) repel each other with a force of Jl 01 2 (21td )

55. When an unpolarized light of intensity 10 is incident on a polarizing sheet, the intensity of the light which does not get transmitted is :

(a) ~ 10 (b) ~ 10

(c) zero (d) 10

56. A charged ball B hangs from +

a silk thread S, which makes +

an angle e with a large +

charged conducting sheet P, P +

as shown in the figure. The +

surface charge density (J of +

the sheet is proportional to : +

(a) cos e (b) cot a +

(e) sin e (d) tan a +

+

57. Two point charges

+ 8 q and - 2 q are located at x == 0 and x = L respectively. The location of a point on the x-axis at which the net electric field due to these two point charges is zero is :

(a) 2L (b) !:_

4

Ce) 8£ (d) 4£

58. Two thin wire rings each having a radius R are . placed at a distance d apart with their axes coinciding. The charges on the two rings are + q and - q. The potential difference between the centres of the two rings is :

(a) ___9!_ . 4m:od2

(b) q [1 1 1

21tEo R - ~R2 + d2

(c) zero

Cd) ~[_! - 1 1 4n:Eo R ~R2 + d2

59. A parallel plate capacitor is made by stacking n equally spaced plates connected alternatively. If the capacitance between any two adjacent plates isC, then the resultant capacitance is:

(a) (n -l)C (b) en + l)C

(e) C Cd) nC

B

60. When two tuning forks (fork 1 and fork 2) are sounded simultaneously, 4 beats per second are heard. Now, some tape is attached onthe prong of the fork 2. When the tuning forks are sounded again, 6 beats per second are heard. If the frequency of fork 1 is 200 Hz, then what was the original frequency of fork 2 ?

(a) 200 Hz (b) 202 Hz

(c) 196 Hz (d) 204 Hz

61. If a simple harmonic motion is represented by

d2x 0·· . d i

dt2 + a X = , Its time peno IS:

(a) 2ll (b) ~

a JO.

ec) 2na Cd) 21t.,ft;

62. The bob of a simple pendulum is a spherical . hollow ball filled with water. A plugged hole near the bottom of the oscillating bob 'gets suddenly unplugged. During observation, tiU water is coming out, the time period of oscillation would:

(a) first increase and then decrease to the original vall!e

(b) first decrease and then increase to the original value

(c) remain unchanged

(d) increase towards a saturation value

63. An observer moves towards a stationary source of sound, with a velocity one-fifth of the velocity of sound. What is the percentage increase in the apparent frequency?

(a) Zero (b) 0.5%

(c) 5% (d) 20%

64. If 10 is the intensity of the principal maximum in the single slit diffraction pattern, then what will be its intensity when the slit width is doubled?

(a) 210 (b) 410

(c) 10 (d) 10

2

65.· Two concentric coils each of radius equal to 21t em are placed at right angles to each other. 3 A and 4 A are the currents flowing in each coil respectively. The magnetic induction in Wb(m2 at the centre of the coils will be :

ijJ.o == 4n:)( 10-7 Wb( Am)

(a) 12 x 10-5 (b) 10-5

ec) 5 x 10-5 Cd) 7 x 10-5

66. A coil of inductance 300 mH and resistance 20 is connected to a source of voltage 2V. The current reaches half of its steady state value in :

(a) 0.05 s (b) 0.1 s

ec) 0.15 s (d) 0.3 s

67. The self-inductance of the motor of an electric fan is 10 H. In order to impart maximum power at 50 Hz, it should be connected to a capacitance of: (a) 4 ~F

(c) 1 ~IF

(b) 8~F (d) 21lP

68. An energy source will supply a constant current into the load, if its internal resistance is :

(a) equal to the resistance of the load

(b) very large as compared to the load resistance

(e) zero

(ci) non-zero but less than the resistance of the load

69. A circuit has a resistance of 12 0 and an impedance of 15 Q. The power factor of the circuit will be :

(a) 0.8 (c) 1.25

(b) 0.4 (d) 0.125

70. The phase difference between the alternating current and emf is 71./2. Which of the following cannot be the constituent of the circuit?

(a) C alone (b) R, L

(c) L, G Cd) Lalone

71. A uniform electric field and a uniform magnetic field are acting along the same direction in a certain region. If an electron is projected along the direction of the fields with a certain

(a) its velocity will decrease (b) its velocity w!ll increase

ec) it will turn towards right of direction of motion

(d) it will tum towards left of direction of motion

72. A charged particle of mass m and charge q travels on a circular path of radius r that is perpendicular to a magnetic field B. The time taken by the particle to complete one revolution is :

(a) 2nmq B

(c) 2nqB m

(b) 2rr.q2B m (d) 2rr.m qB

73. In a potentiometer experiment the balancing with a cell is at length 240 em. On shunting the cell with a resistance of 2 n, the balancing length becomes 120 em'. The internal resistance of the cell is :

(a) 10 (c) 4Q

(b) 0.5 o (d) 20

74. The resistance of hot tungsten filament is about 10 times the cold resistance. What will be the resistance of 100 Wand 200 V lamp, when not in

use? •

(a) 400 (b) 20 Q

(c) 400n (d) 200n

75. A magnetic needle is kept in a non-uniform

magnetic field. It experiences: (a) a torque but not a force' (b) neither a force nor a torque (c) a force and a torque

(d) a force but not a torque

76. The oxidation state of Cr in [Cr(NH3 )4C1zt is: 79.
(a) 0 (b) +1
(c) +2 Cd) +3
77. Which one of the following types of drugs
reduces fever? BO.
(a) Tranquiliser (b) Antibiotic
ee) AntipYl'etic._ (d) Analgesic
78. Which of the following oxides is amphoteric in
character?
(a) Sn02 (b) 8i02 81.
(c) CO2 (d) CaO Which one of the following species is diamagnetic in nature?

(a) Hi (b) H;

(e) H2 Cd) He;

If ('L is the degree of dissociation of Na 1504' the van't Hoff factor (1) used for calculating the molecular mass is:

(a) 1- 20. (c) 1 - ('L

(b) 1 + 2(1 (d) 1 + a

Which of the following is a polyamide?

(a) Bakelite (b) Terylene

(c) Nylon-6Q- Cd) Teflon

82. Due to the presence of an unpaired electron, free radicals are:

(a) cations

(b) anions

(c) chemically inactive (d) chemically reactive

83. For a spontaneous reaction the tJ.G, equiHbriu.tn constant (K) and E;elJ will be respectively:

(a) -ve, > 1, -ve (b) -ve, < 1, -ve .

(c) +ve,. > 1, -ve (d) -ve, > 1, +ve

84. Hydrogen bomb is based on the principle of : (a) arrificial radioactivity

(b) nuclear fusion

(c) natural radioactivity (d) nuclear fission

8S. An ionic compound has a unitcell consisting of A ions at the comers of a cube and B ions on the centres of the faces of the cube. The empirical formula for this compound would be :

(a) A3B (b) AB3

ec) AlP (d) AB·

86. The highest electrical conductivity of the following aqueous solutions is of :

(a) 0.1 M difluoroacetlc acid

(b) 0.1 M fluoroacetlc acid

(c) 0.1 M chloroacetic acid

Cd) 0.1 M acetic acid

87. Lattice energy of an ionic compound depends upon:

(a) charge on the ion and size of the ion (b) packing of ions only

(c)sit,e of the ion only

(d) charge on the ion only

88. Consider an endothermic reaction X -? Y with the activation energies Ef) and El for the backward and forward reactions respectively. in general :

(a) there is no definite relation between Eo

andEf (b) E1,=Ef (c) ·Eb > E f (d) s, < E f

89. Aluminium oxide may be eleetrolysed at 1000 °C to furnish aluminium metal (Atomic mass=Z? amu; I faraday= 96,500 Coulombs). The cathode reaction is

A13+ + 3e- ~ Alo

To prepare 5.12 kg of aluminium metal by this method would require :

(a) 5.49)( 101 C of electricity (b) 5.49)( 104 C of electricity (c) 1.83)< 107 C of electricity Cd) 5.49)< 107 C of electricity

90. The volume of a colloidal particle, Vc as compared to the volume of a solute particle in a true solution Vs,could be :

(a) V.VC '" 103 (b) Vc "" 10-3

S Vs

(c) Vc '" 1023 (c) .Vc "" 1

Vs Vs

91. Consider the reaction: N2 + 3H2 ~ 2NH3 carried out at constant temperature and pressure. If till and 6U are the enthalpy and in rernal energy changes for the reaction, which of the following'expressions .is true?

(a) till > 6U . (b) sn < AU

(c) Mi = su (d) t:Jf = 0

92. The solubility product oJ a .salt having general formula MX2, in water is 4 x·10-12• The concentration of M"l·' ions in the aqueous solution of the salt is :

(a) 4.0)( 10-10 M (b) 1.6x 10-4 M

(c) 1.0)( 10-<1 M (d) 2.0 x 10,6 M

93. Benzene and toluene forrnneady ideal solutions. At 20°C, the vapour pressure of benzene is 75 torr and that of toluene is 22 torr .. The partia! vapour pressure of benzene at 20° C for a solution containing 78 g of benzene and 46 g of toluene in tort is ;

(a) 53.S (b) 37.5

ee) 25 (d) 50

94. Which one of the following statements is not true about the effect of an increase in temperature on the distribution of molecular speeds ill a gas?

(a) The area. under the distribution curve remains the same as. under the lower temperature

(b) The distribution becomes broader

(c) The fraction of the molecules with the . most probable speed increases

Cd) The most probable speed increases

95. For the reaction

2NOz (g) __,.. 2 NO(g)+ 02 (g)

(Ko == 1.8 x 10-6 at 184°C) .

.:

(R :; 0.00831 kJ/(mo\' K)

When It. p and "K c are compared at 184.0 C it is found that:

(a) whether K p is greater than, less than OJ,'" equal to Kc depends upon the total gas pressure

(b) s , =: «,

(e) KI' is less than K,

(d) K p is greater than K;

96. The exothermic formation of ell) is represented by the equation :

Clz(g)+ 3 F2(g)~ 2 C1r:,Cg);.IlHr==':_329 kJ Which of the following will increase the quantity of CIF] in an equilibrium mixture of e12, F2 and cir, ?

(a) Adding r2

(b) Increasing the volume of the container (c) Removing CI2

(d) Increasing the temperature

97. Hydrogen ion concentration in rnol/L in a solution of pH = 5.4 will be :

(a) 3.98 x 10-6 (b) 3.68 x 10-6

ec) 3.88 x 106 Cd) 3.98 x lOS

98. A reaction involving two different reactants C(l,D never be

(a) bimolecular reaction (b) second order reaction (c) first order reaction

(d) unimolecular reaction

99. Two solutions of a substance (nOD electrolyte) are mixed in the following manner.

480 mL of 1.5 M first solution +520 mL of 1.2 M second solution.

What is the molarity of the final mixture?

(a) 2.70 M (b) 1.344 M

(e) 1.50 M (d) 1.20 M

lOO. During the process of electrolytic refining of copper, some metals present as impurity settle as 'anode mud'. These are :

(a) Fe and Ni (b) Ag and Au

(c) Pb and Zn (d) Se and Ag

Calculate i\"'1I0Ae using appropriate molar cond uctances of the electrolytes listed above at infinite dilution in Hp at 2s<'C :

(a) 217.5 Cb) 390.7

(c) 552.7 Cd) 517.2

102. If we consider that 1/6, in place of 1/12, mass of carbon atom is taken to be the relative atomic mass unit, the mass .of one mole of a substance will :

(a) be a function of the molecular mass of the

substance

(b) remain unchanged (c) increase two fold (d) decrease twice

103. In a multi-electron atom, which of the following orbitals' described by the three quantum numbers will have the same energy in the absence of magnetic and electric fields? (A) n == 1, I == 0, m == 0

(B) n == 2, 1= 0, m = 0

(C) n == 2, I:: 1, m = 1

CD} 11 == 3, I '" 2, m ". 1

(E) n = 3, 1= 2, 111 = °

(a) CD) and CE) . (b) (C) and CD)

(b) (B) and eC) Cd) (A) and (B)

104. Based on lattice energy and other considerations which one of the following alkali metal chlorides is expecred to have the highest melting point ?

(a) RbCI (b) KC]

(c) NaCi (d) LiCI

105. A schematic plot of In K.q versus inverse of temperature for a reaction is shown below:

6.0

The reaction must be :

(a) highly spontaneous at ordinary temperature (b) one with negligible enthalpy change

(c) endothermic

(d) exothermic

106. Heating mixture ofCuzO and CUzSwii! give

(a) Cu2S03 (b) CuD + CuS

(c) Cu + 503 (d) co + S02

107. The molecular shapesofSF4, CF4 and XeF" are: (a) different with 1, 0 and 2 lone pairs of electrons on the central atom, respectively (b) different with 0, 1 and 2 lone pairs of electrons on the central atom, respectively (e) -the same with 1, 1 and 1 lone pair of

electrons on the central atoms,

respectively

(d) the same with 2, 0 andj, lone pairs of electrons on the central atom, respectively

108. The disperse phase in colloidal iron CUI} hydroxide and colloidal gold. is positively and negatively cbarged, respectively. Which of [he following statements is not correct?

(a) Coagulation in both sols can be brought about by electrophoresis

(0) Mixing the sols has no effect

(e) Sodium sulphate solution causes

coagulation in both sols

. (d) Magnesium chloride solution coagulates; the gold sol more readily than the iron (III) hydroxide sol.

IU9. The number of hydrogen atom(s) attached to phosphorus a tom in hypophosphorous acid is :

(a) three (b) one

(e) two (d) zero

110. What is the conjugate base ofOH- ?

(a) 02- Cb) 0-

Cc) H20 Cd) O2.

111. Heating an aqueous solution of aluminium chloride to dryness will give:

(a) AI(OH)CI2 (b) AiZ03

(c) A12CI{, Cd) AICIJ

112. The correct order of the thermal stability of hydrogen halides CH - X) is :

(a) HI > HCl < HF > HBr

(b) He] -c HF > HBr < HI

(c) HF > HCl > HBr > HI

Cd) HI > HBr > HCI > HF

113. Calomel (Hg 2C12) on reaction with ammonium hydroxide gives:

(a) HgO

Cb} Hg;iO

(c) NH2-Hg-Hg---Cl Cd) HgNHP

114. The number and type of bonds between two carbon atoms in calcium carbide are:

(a) two sigma, two pi

(b) tWO sigma, one pi

(c) one sigma, two pi

(d) one sigma, one pi

115. Theoxidation state of chromium in the final product formed by the reaction ben ...... een KI and acidified potassium dichromate solution is :

(a) +3 Cb) +2

ec) +6 (d) +4

116. In silicon dioxide :

(a) there are double bonds between silicon and oxygen atoms

(b) silicon atom is bonded to two oxygen atoms

(c) each silicon atom is surrounded by two oxygen atoms and each oxygen atom is bounded to two silicon atoms

(d) each silicon atom is surrounded by four oxygen atoms and each oxygen atom is bonded to two silicon atoms

117. The lanthanide contraction is responsible for the factthat :

(a) Zr and Zn have the same oxidation state Cb) Zr and Hf have about the same radius (c) Zr and Nb have similar oxidation state (d) Zr and Y have abour the same radius

118 .. The IUPAC name of the co-ordination compound K3[Fe(CN)r,J is:

(a) Tripotassium hexacyanoiron (II) (b) Potassium hexacyanoiron (II)

(c) Potassium hexacyanoferrate (1II) (d) Potassium hexacyanoferrate (II)

119. In which of the following arrangements. the order is not according to [he property indicated against it?

(a) Li < Na < K < Rh : Increasing metallic radius

(b) I < Br < F < Cl : Increasing electron gain enthalpy (with negative sign)

(e) B < C < N < 0 : Increasing, first

ionisation enthalpy

(d) AJ3+ < Mg2+ < Na+ < £1- : Increasing

ionic size

120. Of the following sets which one does not conralnisoelecrronic species ?

(a) BO~-, CO~-, NO;

(0) SO~-, co}: NO;

(c) CW, N2, C~-

Cd) PO!_·, so~-, CIO:;

121. 2·methylbutane on reacting with bromine in the presence of sunlight gives mainly:

(a) 1·bromo~3- methylbutane

(b) 2-bromo-3-methylbutanc

(c) 2-bromo-2-methylburane

Cd) 1·bromo-2-methylbutane

122. Which of the following compounds shows optical isomerism?

(a) [Co(CN)6]3.. (b) [Cr(Cp4hJ3~

(e} [ZnC14]2- Cd) [Cu(NH3)4i+

123. Which one of the following cyano complexes would exhibit rhe lowest value of paramagnetic behaviour -?

(a) [eO(CN)6]3- (c) [Mn(CN)6]3-

(b) [Fe(eN)6]3- (d) [er(eN)6]3-

(Atomic no. er = 24. Mn "" 25, Fe"" 26,Co = 27)

124. The best reagent to convert pent-S-en-z-cl into pent-Sen-z-one is :

(a) pyridinium chloro-chromate

(b) chromic anhydride in glacial acetic add (c) acidic dichromate

(d) acidic permanganate

125. A photon of hard gamma radiation knocks a proton out of fiMg nucleus to form:

(a) the isobar of H Na

(b) the nuclide ~:Na

(c) the isobar of parent nucleus (d) the isotope of parent nucleus

126. Reaction of one molecule of HBr with one molecule of 1, 3-butadiene at 40°C gives predominantly:

(a) 1-bromo-2-butene under kinetically

controlled conditions

(b) 3-bromobutene under thermodynamically controlled conditions

(c) Lbromo-z-butene under

thermodynamically controlled conditions (d) 3-bromobutene under kinetically controlled conditions

127. The decreasing order of nuc1cophiIicity among the nucleophiles :

CA) CH3C-O-

II o (B) CH30-

(e) err

°

11 '

CD) H3C-·~S-O~II

o

(a) (C), (8), CAl, CD) (b) (B), (C), CAl, CD) (c) CD), (C), (B), CA) Cd) CAl, (B), ee), CD)

128. Tertiary alkyl halides are practically inert to substitution by SN 2 mechanism because of : (a) steric hindrance (b) inductive effect

(c) instability (d) insolubility

129. In both DNA and RNA, heterocylic base and phosphate ester linkages are at :

(a) e ~ and C~ respectively of the sugar molecule

(b) C~ and c; respectively of the sugar molecule

(e) C; and C ~ respectively of the sugar molecule

(d) C; and e; respectively of the sugar molecule

130. Among the following acids which has the lowest pKa value?

(a) CH3eH2eOOH (b) (CH3hCH-COOtl:

(c) HeOOH Cd) CH3COOH

131. Of the five isomeric hexanes, the isomer which can give two monochlorinated compounds is : (a) 2-methylpentane

(b) 2,2-dimethylbutane (c) 2,3-dimethylbutane (d) n-hexane

132. Alkyl halides react with dialkyl copper reagents to give:

(a) alkenyl halides (b) alkanes

(c) alkyl copper halides (d) alkenes

133. Which one of the following methods is neither meant for the synthesis nor for separation of arnines?

(a) Curtius reaction (b) Wurtz reaction (c) Hofmann method (d) Hinsberg method

134. Which types of isomerism is shown by 2, 3-dichlorobutane ?

(a) Structural (b) Geometric

(c) Optical (d) Diastereo

135. Amongst the following the most basic compound is :

(a) p-nitroaniline (b) acetanilide

ec) aniline (d) benzylamine

136 .. Acid catalyzed hydration of alkenes except ethene leads to the formation of :

(a) mixture of secondary and tertiary alcohols (b) mixture of primary and secondary alcohols (c) secondary or tertiary alcohol

(d) primary alcohol

137. Which of the following is fully fluorinated polymer?

(a) PVC (b) Thiokol

(c) Teflon Cd) Neoprene

138. Elimination of bromine frnm. 2-bromobutane results in the formation of:

(a) predominantly 2-butyne

(b) predominantly t-butene

(c) predominantly 2-butene

(d) equimolar mixture of 1 and 2-butene

139. EQuimolar solutions in the same solvent have: (a) different boiling and different freezing points

(b) same boiling and same freezing points

(e) same freezing point but different boiling point

(d) same boiling point but different freezing point

140. The reaction

o 0

II· II

R -C - X + Nue -+ R -C - Nu + xG!

is fastest when X is : (a) OCOR

(c) NH2

141. The structure of diborane (s._H6) 'contains : (a) four 2C-Ze- bonds and four 3C-2e- bonds (b) two 2C-2e- bonds and two 3C-3e- bonds (c) two 2C-2e- bonds and four 3C-2e- bonds (d) four 2C-2e- bonds and two 3C-2e- bonds

142. Which of the following statements in relation to the hydrogen atom is correct ?

(a) 3s, 3p and 3d orbitals all have the same energy

(b) 3s and 3p orbitals are of lower energy than 3d orbital

(c) 3p orbital is lower in energy than 3d orbital . (d) 3s orbital is lower in energy than 3p orbital

143. Which of the following factors may be regarded as the main cause of lanthanide contraction? (a) Greater shielding of Sd electron by 4f

electrons .

(b) Poorer shielding of Sd electron by 4f electrons-

(c) Effectiveshlelding of one of 4felectrons by another in the sub-shell

Cd) Poor shielding of one of 4f electron by another in the sub-shell

144. The value of the 'spin only' magnetic moment for one of the following configurations is 2.84 BM. The correct one is :

(a) as (in strong ligand field)

(b) d3 (in weak as well as in strong fields) (c) d4 (in weak ligand field)

(d) d4 (in strong ligand field)

145. Reaction of cyclohexanone with

dimethylamine in the presence of catalytic amount of an acid forms a compound. Water during the reaction is continuously removed. The compound formed is generally known as :

(a) an amine (b) an imine

(c) an enamine (d) a Schiff's base

146. p-cresol reacts with chloroform in alkaline medium to give the compound A which adds hydrogen cyanide to form, the compound B. The latter on acidic hydrolysis gives chlral carboxylic acid. The structure of the carboxylic acid is :

CH3 CH3

~CH2COOH

(a) (b)

CH2COOH·1• /.

OH OH

CH3 CH3

~CH(C?~)COOH

(c) Cd) I : .

. /. ,;.

CH(OH)COOH .

OH OH

147. If the bond dissociation energies of XY, X2 and Y2 (all diatomic molecules) are in the ratio of 1 : 1 : o. S and MT J for the formation of XY is-200 kJ mol-i. The bond dissociation energy of X 2 wiJI be :

(a) 400 kJ morl . (b) 300 kJ mol "

(e) 200 kJ mol-l (d) none of these .

148. An amount of solid NH4HS is placed in a flask already containing ammonia gas at a certain temperature and 0.50 atm pressure. Ammonium hydrogen sulphide decomposes to yield NH3 and H2S gases in the flask. When the decomposition reaction reaches equilibrium, the total pressure in the flask rises to 0.84 atm ? The .equilibrium constant for NH4HS decomposition at this temperature is :

. (a) D.H (b) 0.17

(e) 0.18 (d) 0.30

149. An organic compound having molecular mass 60 is found to contain C = 20%, H = 6.67% and N = 46.67% while rest is oxygen. On heating it gives NH3 alongwith'a solid residue .. The solid residue give violet colour with alkaline copper sulphate solution. The compound is ;

(a) CH3CH2CONH2 (b) (NH2)2CO

(c) CH3CONH2 Cd) CH}NCO

150. tl/4 can be taken as the time taken for the

. f d 3 f i

concentration a a reactant to rop to "4 0 Its

initial value. If the rate constant for a first order reaction is k, the tl/'1 can be written as ~

(a) O.75/k (e) O.29/k

(b) O.69/k (d) O.10/k

~~, -----------

1. If C i5 the mid point of AB and P Is any point outside AB, then:

~ ~ ~ ~

(a) PA + PB + PC = 0

~ ~ ~ ~

(b): PA + PB + 2PC= 0

-4- ---) ~

(c) PA + PB = PC

~ ~ ~

(d) PA + PB =2PC

2. tee P be the point (1, 0) and Q a point on the locus y2 == 8x. The locus of mid point of PQ is : (a) x2 - 4y + 2= 0 (b) Xl + 4y + 2= 0 (c)y2+4x+2=0 (d)y2_4x+Z=eO

3. If in a frequency distribution, the Mean and Median are 21 and 22 respectively, then its Mode is approximately:

(a) 24.0 (b) 25.5

ec) 20.5 Cd) 22.0

4. Let R == {(3, 3), (6, 6), (9, 9), (12, 12), (6,12), (3, 9), (3, 12), (3, 6)} be a relation on the set A = {3, 6, 9, 12}. The relation is :

Cal reflexive and symmetric only (b) an equivalence relation

(c) reflexive only

(d) reflexive and transitive only

5. If A2 - A + I := 0, then the inverse of A is :

~)l-A ~) A-I

(c) A (d) A + I

6. If. the cube roots of unity are 1, (0, ol', then the roots of the equation (x -li + 8"" 0, are:

(a) - 1, 1 + 2 W, 1 + 2o}

(b) - 1, 1 - 20), 1 - Zo?

(c) - 1, - 1, - 1

(d) -1, - 1 + Zoo, - 1 - 2w2

. [1 21 2 24

7. lim - sec - + - sec --""2

,,->"" n2 n2 n2 n

n 2 ]

+ .... + zsec 1

,n

equals:

1

(a) '.2 tan 1

1

(c) '.2 cosec 1

(b) tan 1 1

Cd) '2 sec 1.

8. Area of the greatest rectangle that can be 2, y2

inscribed in the ellipse ::'2" + 2"" 0= 1 is :

a b

(a) ~ b

ec) ab

Cb) ..Jab Cd) 2ab

9. The differential equation representing the family of curves y2 '" Zc(x + ·Fe:), where c > 0, is

a parameter, is of order and degree as follows: (a) order 2, degree 2

(b) order 1, degree 3

(c) order 1, degree 1 '

(d) order 1, degree 2

10. ABC is a triangle. Forces P, Q, R acting along IA, IB and IC respectively are in equilibrium, ~he.!! I is the lncentre of f., ABC. Then' P: Q: R is:'

(a) cos A : cos B : cos C

(b) ABC cos "2 : C052: : cos 2'

() . A , B . C c sm"2: sm 2' : sm 2'

(d) sin A: sin B : sin C

11. lfrhe coefficients of r th, (r + 1) th and (,. + 2) th terms in the binomial expansion of (1 + y yo are in AP, then m and r satisfy the equation:

(a) m2 -mC4r-l)+ 4r2 + 2=0

(b) m2 - m(4r+ 1)+ 4r2 - 2= 0

(c) m2-m(4r+I)+4r2+2=O

(d) m2 - m(4,. -1)~· ·4,-2 - 2= 0

12. In a triangle PQR, L R "" %. If tan (%) and tan (%) are the roots of a,x2 + bx + c .. 0, a" 0, then:

(a) b= a + c ec) c = a + b

(b) b = c

(d) a == b + c

13. If the letters of the word SACHIN are arranged in all possible ways and these words are written_ out as in dictionary, then the word SACHIN appears at serial number:

(a) 602, (b) 603

(c) 600 (d) 601

6

14. The value of sOC4 + 1: 56 -'C3 is :

r -1

(a) 56C4 (b) S6C3

(c) 55C3 (d) 5SC4

15. If A "" [~ ~J and I = [~ ~l then which one of the following holds for all n 2'; 1, by the principle of mathematical induction?

(a) N =2n-1A+(n-I)I

(b) An"'llA+(n-l)I

(c) A"=2"-lA-(n-l)I

Cd) A"""rtA-(n-I)I

16. If the coefficient of x7 in [ ax2 + -( b~) r 1 equals

the coefficlent of x-~ in[ ax - (~2) r 1, then a and b satisfy the relation: a

(a) ab = 1 (b) b = 1

(c) a+ b=l (d) a-b==l

17. Let f: (-1, 1)~ B, be a function defined by I(x) == tan -1 ~, then f is both one-one

1- x

and onto when B is the interval:

(a) (-~,~) (b) [-~. ~J

(e) [ 0, ]) Cd)_ (0, ~)

18. If Zj and 22 are two non-zero complex numbers such that 1 21 + z21 c:: 1 211 + 1 221. then

arg ZI - arg 22 is equal to :

(a) - 2: (b) 0

2

(c) - 1t

Cd) ~ . 2

,-" - -' - z

19. If w == ---1-, and I wI = 1, then z lies on :

z - 3l

(a) a parabola ee) a circle

(b) a straight line (d) an ellipse

20. If a2 + b2 + c2 = - 2 and

1 + a2x (1 + b2)x f(x) = (1 + al)x 1 + b2x (1 + al)x (1 + b2)x

(1 + c2)x (1 + c2)x , 1 + c2x

then ICx) is a polynomial of degree:

(a) 2 (b) 3

(e) 0 Cd) 1

21. The system of equations ax+y+z==a-] x+'ay+z",a-I x+Y+Ctz=a-l

has no solution, if a is :

(a) 1 (b) not - 2

(c) either ---: 20r 1 (d) - 2

22. The value of a for which the sum of the sq uares of the roots of the equation x2 - (a - 2)x - a-I'" 0 assume the least value is:

(a) 2 (c) °

(b) 3 Cd) 1

23. If the roots of the equation x2 - bx + c'" 0 be two consecutive integers, then b2 .; 4c eq uals :

(a) 1 (c) 3

(b) 2 (d) - 2

24. Suppose jex) is differentiable at x '" 1 and lim -hI f{1 + h) == 5, then t: (1) equals:

h~O _ .

(a) 6 (b) 5

(c) 4 (d) 3

25. Let / be differentiable for all x. If f (1) = - 2 and I' (x) <: 2for x E(l, 6], then:

(a) /(6) = 5 (b) f(6) < 5

(c) 1(6) < 8 (d) f(6) 2: 8

26. If I is a real-valued differentiable function satisfying I fex) - fey) Is (>; - yi, x, y E R

and 1(0) = 0, then 1m equals:

(a) 1 (b) 2

(e) 0 Cd) -1

If x is so small that x3 and higher powers of x may be neglected, then

(1' + X)3/2 - (1 + ~ x r --------:--:",..--- may be approximated (1- x)1/2

27.

as:

x 3 2 (a) -- - x ,2 8

(c) 3x + l x2 8

3 2 (b) - - x 8

3 2 Cd) 1 - - x 8

.28. If x = L

'"

a.",. y::: L b", z = :E c"

~;O "~O

n;;-O

where a, /;J, c are in AP and

I a! < 1, I bl < 1 I c] <1, then x, y, z are in :

(a) HP

(b) Arirhmetico-Geometric Progression (c) AP

(d). GP

29. In a triangle ABC let LC '" 'It / ~, if r is the inradius and R is the circumradius of the triangle ABC, then 2(r + R) equals:

l.a)c+Q (b)a+b'1"c

(c) a + b (d) b + C

30. If cos" X - cos " ~ = n, then

4x2 - 4.xy cos a + y2 is equal to :

(a) - 4 sin2 a (b) 4 sin2 a

(c) 4 (d)2sin2a

31. If in: a MEC, the altitudes from thevertices A, B, C on. opposite sides are in HP, then sin A, sin B, sin C are in :

(a) HP

(b) Arithmerico-Geometric Progression (c) AP

(d) GP .

32. The normal to the curve

x '" a (cos e + e sin 0), y '" a (sin e - 8 cos 0) at any point . 0' is such that:

(a) it is at a constant distance from the origin (b) it passes through (a 11/2, - 0)

(c) it makes angle 1(/2+ e with the x-axis (d) it passes through the Origin

33. A function is matched below against an interval where it is supposed to be increasing. Which of the following pairs is incorrectly matched?

Interval Function

(a) (- =. - 4J x3 + 6x2 + 6

(b) (- (I), ~ ] 3~2 - 2x + 1

(c) [2, 00) (d) (- m, t:O)

2x3 _ 3x2 - 12x + 6 x3 - 3x2 ... 3x + 3

34,

Let Cl and {3 be the distinct roots ax2 + bx + C,;, 0, then .

1 - cos (ax2 + bx + c) . 1

lim 2 IS equa to:

X-->a (x-a)

(a) ~ (a - {3)2

(e) °

a2

(b) - - (u-j3l 2

a2

(d) - (a. -l3i

2 .

35.

If x: == y (log y - log x + 1) , then the solution of the equation is :

(a) log (~) = cy (b) log (~) ==~x

(e) XIOg(~) =cy Cd) YIOg(~)=CX

The line parallel to the x-axis and passing through the intersection 'of the lines ax + 2by + 3b == 0 and bx - 2ay - Sa == 0, where Ca. b) ;J' (0, 0) is :

.(a) above the x-axis at a distance of (2/3) from it.

Cb) above the x-axis at a distance of (3/2) from it,

C c) below the x-axis at a distance of (2/3) from it.

Cd) below the x-axis at a distance of (3/2)

from it.

A spherical iron ball 10 em in radius is coated with a layer of ice of uniform thickness that melts at a rate of 5'0 em3/min. When the thickness of ice is 15 em, then the rate at which th€ thickness of ice decreases, is :

Ca) ~ ern/min (b) S!1t em/min

(e) l~n ern/min Cd) 3~-:;:;cm/min

f { (log·x _1)}2 dx is equal to : 1 + (log xi

x xc

(a) + c (b) --2 + C

(iogxi+l l+x

36.

37.

38.

(d) log x + c

(log x)2 + 1

39. Let f: R --) R be a differentiable function having 1(2) == 6,./ ' (2) = ( 4~) ,

f''f(X) 4t3

Then lim --2 dt equals:

x_,.2 6 _x-

of

(a) 18 (c) 36

(b) 12 (d) 24

40.

Let f (x) be a non-negative continuous function such that the area bounded by the curve y = j(x). x-axis and the ordinates x = Jt/4and x=J3>n/4 is

( J3 sin t3 + ~ cos f3 + .J2 J3)- Then 1 C~) is ;

41.

(b) (1 - * - /2)

ec) (* -.[2 + 1) Cd) (% -I- -Ii - 1)

1:1 1 3

Ifl1 '" JZ~"dx, 12", J2'" dx,

o ()

2 2 :I 3

13'= fZ< dx and I, = fZ~ dx, then:

1 1

(a) [3> 14 (b) 1:! '= 14

(c) II > 12 (d) I'l > II

( 7t r.-)

!; (a) 1 - '4 + "i 2 .

42. The area enclosed between the CUIVC y '""' log, (x + e) and the co-ordinate axes is :

(a) 4 (b) 3

(e) 2 Cd) 1

43. The parabolas y2 "" 4x and Xl '" 4y divide the square region bounded by the lines x ;= 4, y ~ 4 and the co-ordinate axes. If 51' SOb 51 arc respectively the areas of these parts numbered from top to bottom, (hen 51 : 52 ; S.:1 is :

(a) 1: 1 : 1 (b) 2: 1 : 2

(c) 1 : 2 : 3 (d) 1 ; 2 : 1

44. If the. plane 2ax - 3o.y + 4az + 6 =' 0 passes through die mid point of the line joining the

centres of the spheres

Xl + yl + z2 + 6x - 8y - 2z =' 13 and

x2 + y2 + Z2 -lOx + 4y - 2z = S, then uequals :

(a) 2 (b) - 2

ee) 1 Cd) - 1

45. The

between

the

distance

-l-' • •

plane r ,(i + Sj + k) '" 5 is :

( ) 10 (b) 2

a 3 10

ec) ..!Q_ (d) 10

313 9

-}

46. For any vector a, the value of

(1 x iy + (~ x Jl + a x k)2 is equal to :

-)2 (a) 4a

-)2 (e) a

(b) 2i,2 (d) 33:)2

47. If non-zero numbers a, b, [are in HP, then the

iah u x y 1. 0 I

straignt me a + b + C = a ways passes

through a fixed point. That point is :

(a) (1) -~) (b) (1, - 2)

(c) (- 1, - 2)

Cd) (-1 .• 2)

lil)e

48. If a vertex of a triangle is (1, 1) and the mid points of two sides through this venex are (- 1, 2) and (3, 2) , then the centroid of the triangle is : .

(a)G,~) (b)(l,~)

(c) (- j, ;)1 Cd) (-1, ~i

, .I

49. If the circles x2 + y2 + 2ax -t- cy ... a"" 0 and x2 + y2 _ 3ax + dy - 1 '" 0 intersect in two distinct points P and Q, then the line 5x + by - a ~ 0 passes through P and Q for : (a) exactly twO values of Q

(b) infinitely many values of a (c) no value of a

(d) exactly one value of a

SQ. A circle touches the x-axis and also touches the circle with centre at (0, 3) and radius 2. The locus ofthe centre of the circle is :

(a) a parabola (b) a hyperbola

(c) a circle (d) an ellipse

51. If 01 circle passes through the point (a, b) and cuts the circle x2 + y2 =; l' orthogonally, then

the equation of the locus of its centre is ;

(a) lax + 2hy - (a2 + b2 + p~) == o·

(b) x2 + y2 - 2ax - 3by + (el'" - b" -l) '"" 0 (c) 2ax + 2by - Ca2 - b~ + 1'2) = 0

Cd) x2 -r- y2 - 3ax - 40y + (£12 +- b2 - p<) ca 0

52. An ellipse has OB as semi minor axis, F and F'· its foci and [he angle F13F' is a right angle. Then the eccenrricuy of the ellipse is :

(a) -~ (b) I_

n 4

(e) ~ (d) 32

53. The locus of a point pen, (3) moving under the condition that the line y := a x + n is a tangent ~ l

to the hyperbola -\ - y 2 = 1 IS : a b

(a) a hyperbola (b) a parabola

(c) a circle (d) an ellipse

54. If the angle 0 between" the line

x+1 Y-l z-2 1-="-2-=-2-

2x - y + Jf..z -1- 4 = 0 is such that sin 0 = ~. The

and

the

plane

value of 1. is : (a) _i

3

3

(c)

5

3 (b) _.

4

(d) ~ 3

55. The angle- between the lines 2x =' 3y =: - z and 6x = - y =' - 4z is :

(a) 30e (b) 45°

(c) 90° (d) 0°

56. Let A and B be two events such that

-- 1 1 - 1

peA u B) = "6' peA n B) "'"4 and peA) ="4 '

where A stands for complement of event A.

-, Then events A and Bare:

(a) mutually exclusive and independent (b) independent but not equally likely (c) equally likely but not independent

· (d) equally likely and mutually exclusive

57. Three houses are available in a locality. Three persons apply for the houses. - Each applies for one house without consulting others. The probability that all the three apply for the same house, is: (a) 7/9 (c) 1/9

(b) 8/9 (d) .2/9

58.

A random variable X has Poisson distribution with mean 2. Then P(X :> 1. 5) equals:

(a) 1_ (b) l-~

e2 ~

2

ec) 0 (d) 2""

e

Two points A and B move from rest along a straight line with constant acceleration f and t respectively. If A takes ni sec more than B and describes 'n' unit more than B in acquiring the same speed, then:

(a) (f' - f)n =! ff' m2

2

(b) ~ (f + f')m = ff' n2 (c) (f + f')m2 :.Ifln· (d) (f - f')m2 -.; ff'n

A lizard, at an initial distance of 21 em behind

· an insect, moves from rest with an acceleration

· of 2 Cm/S2 and pursues the insect which is crawling uniformly along a straight line at a speed of 20 cm/s. Then the lizard will catch the insect after:

59.

60.

(a) 24 s (c) 1 s

(b) 21 s Cd). 20 s

61.

The resultant R of two. forces acting on a particle is at right angles to one of them and its magnitude is one third of the other force. The ratio of larger force to smaller one is :

(a) 3:2.fi :(b).-3:2

(c) 3: v'2 Cd)' 2: 1

62. Let ~;; i - k, b ~ xi + ] + (1- x)k and

-4> "' - • -+ -:l'-jo

C ==yi+xj+(l+x,..y)k. Then [a 1) c]

depends on:

(a) neither x nor y' (b) both x and y

(c) only x (d) only y

63. Leta, b and cbe distinct non-negative numbers.

If the vectors ai + aj + ck, i + k and

ci + cJ + bi{ lie in a plane, then cis:

(a) the harmonic mean of a and b

(b) equal to zero

(c) the arithmetic mean of a and b (d) the geometric mean of a and b

64. If 1. b, ~ are non-coplanar vectors and A, is a real number, then

[1..(1 + b) Alb 1..1]<=:[1 6 + 1 6]

for:

CaJ exactly two values of A (b) exactly three values of A (c) no value 00.

(d) exactly one value of A

65.

A and B are two like parallel forces. A couple of moment H lies in the plane of A and B and is contained with them. The resultant of A and B after combining is displaced through a distance:

H (a) A-B

11 (c) A+ B

H

(b) 2(A + B)

Cd) '2H A-B

66.

The sum of the series

1 1 1 .

1+ 4.21+ 16.41+ 64.61+ ... 0015:

(a) e+ 1 (b) e -1

zJe 2..Je

(c) e+l Cd) e-l

.Je ..Je

67.

Let Xl' X2' ••.•• , XII be n observations such that Exl == 400 and :E x, = 60. Then a possible value

of n among the following is :

(a) 12 (b) 9

(e). 18 (d) 15

A particle is projected from a point 0 with velocity u at all. angle of 60° with the horizontal. When it is moving in a direction at right angle to its direction atO, then its velocity is given by :

68.

(a) ~ J3

(b) 2u 3

u Cd) ~

(c) 2. 3

69. If both the roots of the quadratic equation x2 - 2kx + k2 + k - 5 = 0 are less than 5, then k lies in the interval:

(a) [4, 5] (b) (- 00,4)

(c) (6, ex» (d) (5, 6]

70. If al• 0.2• 0.:1, •..•. , a" •... are in GP, then the determinant

'I log G" log 0.11_ 1 log an, 2.

d'" log a,,_ 3 log a" .. 4 log a". s

log all ~ 6 log ~ + 7 log a" ... 8

is equal to:

- (a) 2 (b) 4

(c) 0 (d) I

71. A real valued function fex) satisfies the functional equation

f(x - y) = fex) fey) - tea - x) f(a + y) where a is a given constant and teO) = 1, f(m - x) is equal to:

(a) f(- x) (b) fCa) + fea - x)

(c) I(x) (d) - I(x)

72. If the equation

all xR + u,,_IXn-1 + ..... + CZJx '" 0,

0.1 "F- 0, Tl 2: 2, has a positive root x '" 11, then the equation

na"xn-l + (n - l)an_ ,x,j- 2 + ... + ~ = 0 has a positive root, which is :

(a) equal to 11

(b) greater than or equal to 11 (c) smaller than u

(d) greater than a

][ 2

73. The value of J cos x dx, a> 0, is : 1 + a"

.. "

(a) 27t-

(b) 2!_

a

7t

ec) - 2

(d) an

74. The plane x + 2y - Z = 4 'CUtS the sphere x2 + y2 + Z2 - X + Z - 2 = 0 in a circle of

radius :

(a) .J2 (b) 2

(c) 1 (d) 3.

75. If the pair of lines ax2 + 2{a + b),xy 1- by2 = 0 lie along diameters ofa circle and divide the circle into four sectors such thin the area of one of the sectors is thrice the area of another sector, then:

(a) 3a2 + 2ab + 3h2 :. 0 (b) 3c? + lOab + 3bz = 0 (c) 3a2 - 2ab +-3b2 '" 0 (d) 3a2 - lOab + 3b2 = 0

/llNSWERS

I •• PHYSICS AND CHEMISTRY
1. (d) 2~ (d) 3. (a) 4. ec) 5. (d) 6. (c) 7. ec) 8. «)
9. (d) 10. , (e) 11. (d) 12. (a) 13. (b) 14. (d) 15. (d) 16. (a)
17. (c) 18. (d) 19. (b) 20. (c) 21. (a) 22. (b) 23. (a,d) 24. (e)
25. (cl 26. (d) 27. (b) 28. Cd) 29. (e) 30. Cb) 31. Ce) 32. (a)
33. (c) 34. (e) 35. (a) 36. (b) 37. (d) 38. (a) 39. (b) 40. (a)
41. (a) 42. (a) 43: (c) 44. ee) 45. eel 46. (b) 47. (d) 48. (b)
49. (b) 50. (b) 51. (d) 52. (d) 53. (a) 54. (a) 55. (a) 56. (d)
57. (a) 58. (b) 59. (a) 60. ec) 61. (b) 62. (a) 63. (d) 64. (c)
65. (e) 66. (b) 67. (e) 68. (e) 69. (a) 70. Cc) 71. (a) 72. (d)
73. (d) 74. (a) 75. (c) 76. (d) 77. (c) 78. (a) 79. eel 80. (b)
81. (c) 82. (d) 83. (d) 84. (b) 85. (b) 86. (a) 87. (a) 88. .(d)
89. (d) 90. (a) 91. eb) 92. ee) 93. (d) 94. (e) 95. (d) 96. (a)
97. (a) 98. (d) 99. (bl 100. (b) 101. (b) 102. (b) 103. (a) 104. (e)
,105. (d) 106. (d) 107. (a) lOB. ee) 109. (c) 110. (a) 111. (b) 112. (e)
113. (d) 114. (e) 115. (a) 116. (d) 117. (b) 118. (c) 119. (c) 120. (bl
121. (c) 122. (bl 123. (a) 124. (b) 125. (b) 126. (e) 127. (b) 128. (a)
129. (a) 130. (e) 13l. (el 132. (b) 133. (b) 134. (el 135. (d) 136. eel
137. ' (el 138. ee) 139. (b) , 140. (d) 141:, (d) 142. (a) 143. (d) 144. (d) .
145. (e) 146. (el 147. (d) 148. (a) 149. (b) 150. (e)
* No option is matching
"" MATHEMATICS
.
L (d) 2. (d) 3. (a) 4. (d) 5. (a) 6. eb) 7. (a) 8. (d)
9. (b) 10. Cb) 11. (b) 12. (c) 13. Cd) 14. (a) 15. (d) 16. (a)
17. (a) 18. (b) 19. (b) 20. (a) 21. (d) 22. (d) 23. (a) 24. (b)
25. (d) 26. ee) 27. (b) 28. (a) 29. ee) 30. eb) 31. (e) 32. (a, e)
33. Cb) 34. (d) 35. (b) 36. (d) 37 .. (e) 38. (a) 39. (a) 40. (a)
41. (e) 42. (d) 43. (a) 44. (b) 45. ee) 46. (b) 47. (b) 48. (b)
49. (e) 50. (a) 5l. (a) 52. (d) 53. (a) 54. (d) 55. ee) 56. (b)
57. ee) 58. (b) 59. (a) 60. (b) 61. (a) 62. (a) 63. (d) 64. (e)
65. (e) 66. (a) 67. (e) 68. (a) 69. (b) 70. (e) 71. (d) 72. (c)
73. (e) 74. (e) 75. (a) HINTS fI SOLUTIONS

~~-----

1. A .projectile can have same range if angles of projection are complementary i.e., e and (900 - 8). Thus, in both cases:

2u sin 8 t,"'---

g

2u sin (900 - 8)

t 2 == --___:- _ __:.

g

2u cos 8

... (i)

... (ii)

s

From Eqs, (i) and (ii)

4u2 sin 0 cos 0

tit 2 == 2

g

. 2u2 sin 21}

tjt2 '" 2

g

2 u2 sin 28

g g

Hence, tit 2 0: R

2. Since to is constant, v would also be constant. So, no net force or torque is acting on ring. The force experienced by any particle is only along radial direction, or we can say the centripetal force.

The force experienced by inner part, FI = mro2R, and the force experienced by outer

part, F2 == molR2

FI R, F2 = R2

3. When friction is absent

aJ =g sin e 1 2 Sl=-za1tl

···ei)

When friction is present a2 == g sin 8 - ~ k g cos 8

1 2 52 == 2: a2!2

... (ii)

or

aJ '" n2a1

a2 g sin e - 1-1 k g cos {) 1

al == g sin {) = n2

g sin 45° - 11k g cos 45° 1

g sin 45° n2

1 l-I-Ik"'n2

1 [.I.k =1-n2

or

or

or

or

4. According to work-energy theorem, W=6K=0

(": Initial and final speeds are zero) :. Work done by friction + work done by gravity / ==0 - J.I mg cos~)~ + mgl sin ~ == 0

or

e__ cos $ == sin $ 2

.. I..l = 2 tan $

5. According to work-energy theorem, W=M<

Case I: - F x 3 == ~ m(VoJ2 - ~ mVo2

. 2 2 2

where, F is resistive force and Vo is initial speed.

Case II : Let, the further distance travelled by the bullet before coming to rest is s.

-f! (3+ s)=Kt - Ki;; -~ mv8

1 2 1 2

or . - 8 mvo (3 + 5) == - "2 mvo

1

-(3+ s);d 4

~+.!.=1 4 4

or

or

s =1 em

6. I", mr2

.. [1] == [Me]

-t ~ and T = moment of force = r x F

7, Given c = ax:2 + bx

Differentiating w.r.t. t ,

de dx dx

de = 2ax dt + b"di:"

dx 1

v = dt' == (Zax -+ b)

Again differentiating. W.r.t. t

. d2x -2a dx del = (2ax + b)2' dr

d2x'

f "'" de2

-1 2a

'" (2 ax + bi . (2ax + b)

-2a J=~-';___'"

(2ax + bi

f == - Zav 3

or

8. The velocity time graph for the given. situation can be drawn as below. Magnitudes of slope of OA=f

y

v v (mls)

A

and slope of Be ;; {

f

v == Jtt ="2'2

.. t2 == 2tl

In graph area of t:.OAD gives

distances,

... (i)

Area of rectangle ABED gives distance travelled in time c.

52 = (ftl)t Distance travelled in time r 2

1 f 2

= 53 =="2 "2 (2c 1 )

Thus;

SJ + 52 + 53 = 155 5 + (fil)e + fr? = 155 S + (frI)t +·25 = 15S

(ftl)t = 125 From Eqs. (i) and (ii), we have

125 (ftl)t

5=1 .

2 (fti)tl

t

t1 =6'

... (ii)

From Eq. (i), we get

.. S=~ feed

1 (t)2 1 2

5='2/6" =72ft

Hence, none of the given options is correct. -t -;

9. VI =+ 51

N

Av

W--------~--------E

.... -v,

s

-» ~ --+ .... .....

8V=V2-Vl",,5J-5i

'16~1=sJ2

For direction,

5

tan u = - 5 "'-1

Average acceleration is Jz. ms-2

towards

north-west.

10. Parachute bails out at height H from ground.Velocity at A

v = ..jr>2{~h __ = .J2 x 9.8 x 50

'" .)980 rn/ s The velocity at ground Vt '" 3 mI s Acceleration = - 2 m/ S2

(gjven) (given)

1

H

I

Ground

y,2 _ y2 H - h '" _....=..:.L 2x2

980 - 9 4

= 971 ~ 242.75 4

H '" 242.75+ h

= 242.75 + 50"" 293 m

11. In the frame of wedge. the force diagram of bl~k is shown in figure. From free body diagram of wedge.

N

mg coso; +masina mg

mg sina.

For block to remain stationary, ma cos Ct = mg sin Ct a= g tan a

12. According to conservation of energy.

or

mgH = ~ mv2 + mgh2 1 2

mg(H - h2)= 2. mv

v ;; ,j2g(100 - 20)

or

13.

v == ,}2 x 10 x 80 =; 40 m/ s

Since, the accelera tion of centre of mass in both the cases is same equal to g. So the centre of mass of the bodies Band C taken together does not shift compared to that of body A.

The mass of complete (circular) disc is

14.

L.~

o :

. ,

, .

\. ;'

-, /

.... ,....... ~'" ",;,'"

" -

.......... - .. _----_ .......

M+M=2M

The moment of inertia of disc about the given

axis is

2Mr2 1=-2-

15.

=Mr2

Let. the moment of inertia of semicircular disc is 11' The disc may be assumed as combination of two semicircular parts.

Thus, It '" I-II

I Mr2

., II =''2=2

Given: m = 0.3 kg, x = 20 em, and bd5N/m

F =-kx

and

... (i) .,.(li)

F=ma rna = -Toe

15 20 10-2

a=- 0.3 x x

a=_15x2=_10m/52 3

.. Initial acceleration a = 10 m/ 52

or

16. Momentum would be maximum when KE would be maximum and this is the case when

total elastic PE is converted to KE. According to conservation of energy

.!kL2=.!Mv2

2 2

ke = (Mv/

M

MkI} =: p2

p=L.JMk

17. In x-direction

mU1 + 0", 0 + mv"

~ vx =v

In y-direction

0+ 0", m(JJ) - mvy

Vy Aller Collision

v ,Vy '" ...fj

.'. Velocity of second mass after collision

v';; (2.._)2 + v2 '" ~i v2

13 3

, 2 v"'...fjv

18. Water fills the tube entirely in gravity less

19.

condition.

Energy stored in wire

1 . 1

'" "2 stress x strain x vo ume

stress

and Young's modulus = --.stram

. S

~ stram",y

Energy stored in wire 1 .

Volume "'"2 x stress x stram

1 5 '52

=-5x-=-

2 Y 2Y

s= ~rv;; M =(;nR3)p

4G nR3 g=3R2"P

g = (4C:) p

20.

(p = average density)

~ g oc P or P oc g

mv

21. F=ma=T

( v- 0)

,,:. a = ----y-

Instantaneous power", Fv =mav

y2 100 x100

s = ---- '" -----

2).lkg 2xO.5)(10

"" 100 x 100", 1000 m Sx2

23. Statements (a) and (d) are wrong. Concept of entropy is associated with second law of thermodynamics.

24. For pure translatory motion, net torque about centre of mass should be zero.

22.

--)

Th us, F is applied at centre of mass of system.

(0,21)



• 10 21)

P\' 3

y



25.

. OP '" m1 xO + m2 x I

mJ + m2

where, ml and m2 are masses of horizontal and vertical section of the object. Assuming object is uniform,

· 21

m2 '" 2m} ~ OP '" 3

PC = (1- ~l + l)

= (21- ~l) = ~l gh =s (1- ~h) gd,=g(l-i)

From Eqs, (i) and (ii),

g(i_~h)=g(l_*)

.. . (i)

... (ii)

2h=d

~ ..

GMm 26. U1",,---

r

U. =. _ 6.67 xlO-ll x 100 x 10-2

I 0.1

R=O_1 m

M=100kg

6.67 X 10-11

Ui'" - 0.1

=- 6.67 )(10-10 J - We know

: .. ~ W=/1U

27.

=U, -U;

W == -Ui == 6.67 x10-10 J

"1 CV) +"2 CV2

C y "" _.--:---!.---=-~

n1 + n2

For helium, "1 == 1: = 4 and Yl = ~

16 1 7

For oxygen, "2 = 32 = "2 and h = "5

(-: U, == 0)

Now,

=>

28. ['=1 e-j1 x

l' -I-' x= logy

1 -jJ.. 36 = log 8f

'1 1

-I-' x = og 21

From Eqs, (i) and (ii),

36 310ga)

.7= 1

log "2

x'=12 rom

... (i)

... (ii)

29.

Eg ==hv

h C ( 6.63 x 10-34)( 3 X 108 ) V

""' T ==, 2480)( 10-9 x 1.6)( 10-19 e

=O.5eV

la (1d Y; == (ra)2

12 oP

Y;=(~r

30.

31.

[2 == 411

Now, since number of electrons emitted per second is directly proportional to intensity, so number of electrons emitted by photocathode would increase by a factor of 4.

N "" No (1 - e-). f)

No -N -H

---=e

No _

1 -).f g=e

8:::; eA.'

31n2=).t

). = 3 x 0.693 15

Half-life period

0.693

tll2 == 3)(0.693 xlS

tll2 == 5 min

32.

_ (27F3 3 - (l2SF3 5

'RTe '" ~ x3.6

RTe '" 6 fermi 33. According to the figure

T

2TQ ts
To 3 2
SQ 2SQ S Q1 '" To 80 + ~ To So '" ~ To So Q2"'To (1$0 - So) = To SO

Q3 ='0

W Q1 ~Q2

11 '" QI = Q1

1 Q2 1 2 "" - Q; '" -"3

1

3

34. To measure the radial rate of heat flow, we have to go for integration technique as here the area of the' surface through which heat will flow is not constant.

Let us consider an element (spherical shell) of thickness dx and radius x as shown in figure. Let us first find the equivalent _ thermal resistance between inner and outer sphere.

Resistance of shell = dR == dx 2.

K x47U:

[ l 1

from R = KA where

K --+ thermal conductivity

3S.

r2 -fi

411: K (r,'2)

Rate of heat flow", H

Tl -T2 T, -T2 4-V( ). 'i'2 =-_-==--x , ..... 'i'2 oc--

R ~-~ ~-~

The change in internal energy does not depend 'upon path followed' by the process. It only depends on initial and final states,

Hence, 6U1=6U2"

Here, y "" sin2 rot

36 •.

: '" 2 ro sin rot cos 0) r = 0) sin 2 {Ot

d2 ForSHM, . ~ oc-y

dt

Hence, function is not SHM. but periodic. From the y-! graph, time period is

T=~

ID

38.

Given:

Yl = 0.1 sin (100 n:t -+ })

:, ~l '" VI = 0,1 )(,100 'It cos (100 1tt -+ ~)

or VI = IOn sin (100 7ft -+ j + ~)

or 'VI = IOn sin (100 nt + ~)

and Y2 = 0.1 cos nt

.• ~2 = v2 = - 0.1 sin 7ft '" 0.1 sin (7ft + n)

Hence, phase difference

3$ '" ~1- ~2

= (100 nt + S:) - (nt + n)

= Sn _ 1t (at t = 0)

6 .

=-"6

39. The situation is shown in figure ..

or

40. We know

Y AD ::: 1.22"d

yd D s ].221

10-3 )( 3 X 10-3 "" 1.22 x 5 x 10-7 30

=6] '" Sm

Dmox "" Sm

41.

L '" (P -1) (;1 - '~J "" {1.5 -1)(;1 - ;J

and L = (~8 ~-:m ) (i1 - iJ

... (i)

Thus,

'" 1.6 in

p =~ '" 1.6 =1 D

m fm 1.6

42.

, '" Rhe [1~] '" 0.2Rhc E(2 ->1) '" Me [(Ill - (;)2J

'" Me [~] '" a.75Rhe

Er1->J) ",Rhe[C;p - {11i J

'" - ~ Bhc '" - 0.9 Me

9

Thus, III transition gives most. energy. 1 transition represents the absorption of energy.

43. We know

and

=> Thus,

44. In common base amplifier the input signal is amplified but remain in phase with output signal.

45. Given f "" so Hz

1

.. T '" SO

For full wave rectiJierTI "';= Ibo 11:: 100Hz

/I I ? -I

46. ZX + n ~ Li + He

o 3 2

It implies that A + 1 =7 + 4

=> A == 10

and Z+0",3+2

~ Z=5

10 Thus, it is boron B.

5

47. Full scale deflection current

= 150 rnA =lSmA 10 .

Full scale deflection voltage 150

=-mV =75mV 2

. 75mV

Galvanometer resistance G = 1 smA = SO

Required full scale deflection voltage,

V", 1 x 150 == 150V

Let resistance to be connected in series is R.

Ig R G

~

~------~V'----------

v= 19(R + G)

ISO"" 15 X 10-3 (R + 5)

104", R + 5

R == 10000 -.5 = 9995

=>

48. Weknow

m=zq

1 za:;, q

Zz '" iJl Zl q2

Total charge q = III + qz

.!.",Cb +1

q2 q2

49. The galvanometer shows zero deflection. i.e., current through XY is zero

500n

x

12V

R

As a result potential drop across R is 2 V circuit can be redrawn as

500 ~~

l_

2V

R

50.

12

1 == 500 + R

Voltage across R, V = IR

12

=> 2= 500+ R xR

=> 1000 + 2R = 12R => R = lOOn R.q = Rt + R2 + R

R'~B

E E

R

According to the question,

-(VA - Vs)=E -1 R2 O=E'-IR2 E =[ R2

E ==', ,2E'. R2 Rl ,.. R2 + R

. Rt + R2 + R = 2R2

R '" ~2 - Rl

51.

... (i)

The energy stored in capacitor is lost in form of heat energy:

H", ms AT

... (ii)

52.

From Eqs. (i) and (ii), we have

(1) 2

ms st = "2 GV

V = ~'2m~liT

Relative velocity == v - (- v) == 2 v = ~:

Now, e= ~i

Bidl

e=-

dt

Induced emf

e== 2Elv V2 "v:»:'

VZ H2 = R/2t

H~ '" 2

HI

:::::) Hz = 2H}

54. The force per unit length between the two

wires is

F f.lo 2e '1= 47t'cr

l-I 0 i2 21td

The force will be attractive as current directions in both are same.

55. 1=10 C052 8

Intensity of polarized light 10

='-2

:. Intensity of untransmitted light 10 10

=10 -2= 2

56. Electric field due to a charged conducting sheet of surface charge density 0" is given by E = ~ Evf'T

TcosG

mg

where, &0 = permittivity in vacuum and f'T relative permittivity of medium.

Here, electrostatic force on B QE=QO"

£{)~~r

FED of B is shown in figure. In equilibrium, T cos 0", mg

and Y· '. 0 Q 0"

sm ."'evEr

Thus,

tan 8 a; 0"

or cr 0:: tan e

57. Suppose that a point B , where net electric field is zero due to charges 8q and-2q.

B' .

A x"'L

+8q o

-2q

X=8

x=o

-+ -1 8q~ EBa =--'-1 4 "It Eo a2

-+ 1 + 2q

EpA "'--.-__,~

. 4 It Eo (0 + Li

-+ -+

According to condition E 80 + E £iA '" 0

1 8q 1 2q

410;9 a2 = 41tf-o (a + L)2

2 1

a=a+L

2a+ 2L = a

2L =-a

Thus, at distance 2L from odgin, net electric field will be zero.

58. VA:= potential due to charge + q on ring A. + potential due to charge - q on ring B

+q

-q

= _1_ (!l_.!L) , d1 = ~R2 +d1 41tEo R d,

.= .. ~ [t - JR:+ d') ... Ci)

Similarly, V~ '" 4 ~ Eo [ - t + ~2)

pore:"~:.[*e~:::::) _ 4!~

[~q+ ~R2: d~)

= _1 __ <!. + _1 _ _i _ -.J:_ q

4m:o R 4ltEo R 41tto JR2 + d2

1 . q

- 4ltEo JR 2 + d2

; 2;f~ [f - JRm2: d2 )

59. Each plate is taking pan in the formation of two capacitors except the plates at the ends.

These capacitors are in parallel and n plates form (n - 1) capacitors.

Thus, equivalent capacitance between A andB =(n-l)C. .

60. The frequency of fork 2

= 200 ± 4 = 196 or 204 Hz

Since, on attaching the tape on the prong of fork 2. its frequency decreases, but now the number of beats 'per second is 6 i.e .. , the frequency difference now increases. It is possible only when before attaching the tape, the frequency' of fork 2 is less than the frequency. of tuning fork 1. Hence, the frequency of fork 2 is 196 Hz.

d2x

dt2=-ax ... (i)

61.

We know

d1x 2

Q= dt2 =-(1) x ... (ii)

From Eqs. (i) and (ii), we have «)2 '" 0:

ro=JU 2T1': =..fa.

T _ 2n

-.;;;

or

62.

Spherical hollow ball filled with water

T=2T!,fl; 9

Spherical hollow ball

T2~2T!a

9

Sphericalhollow ball half filled with water

T,= 2T!JI+N 9

and Tl >T2

Hence, time period fir-it increases and then decreases to the original value.

. v 320

GIven: Vo =="5'~ Vo '" -5- = 64 m/s

63.

When observer moves towards the stationary source, then

n' = (v: vo) n

,_(320+ 64)

n - 320 n

. (384)

n ;; 320 n

n' 384 n= 320

Hence, percentage increase

(n. - n) "" (384 - 320 x 100) %

n .320

= (64 x 100) % = 20 % 320

64. I =10 (Si~ er

and

For principal maximum y = 0 (-)=0

Hence, intensity will remain same.

65 B _1-10/2

• P - 2R

B

.= 41t x 10-7 x 4 == 4 X 10-5 Wb/ m2 2xO.02'1t

B - ~o 11

Q-.2R

= 4 'It X 10-7 X 3 =3 x 10-5 Wb/m2 2 x 0.02 'It

.', B =~B~ + B~

= ~(4 x lO-si + (3 x 10-si == 5 x 10 .. 5 Wb/m2

The current at any instant is given by

I=Io(l-e-RtlL) L.R

!J;_ == 10 (l - e-R1.IL) 2

1. = (1 _ « RtIL) 2

e-Rr/L = 1/2

Rt=ln2 2V

L

L 300 X 10-3

.. t =-In 2=--~-·- x 0.693

R ,(

= 150 ;.: 0.693 x 10-3

== 0,10395 s = 0.1 s 67. Given: L =.10 H, f = 50 Hz

66.

For maximum power

Xc =XL

1 -=(J}L roC

C=_l_ (o2L

c= 1

4 n? x so x 50 x 10

C == 0.1 xlO-5 F == 1 ~F

E 68. 1=-. R + r

E

I =' R = constant

where, R = external resistance r ee internal resistance =0

69. Power factor

R =cos~=Z

12 4 ='15='5 =0.8

70. L In a circuit having C alone, the voltage lags 11;

the current by 2'

2. In circuit containing Rand L, the voltage 1t

leads the current by '2 .

3. In I.C circuit, the phase difference between current and voltage can have any value

between 0 to ~ depending on the values of Lande. -

4. In a circuit containing L alone, the voltage 1t

leads the current by 2: .

---}> ---}> ---}>

When E, v and B are all along same

direction, then magnetic force experienced by electron is zero while electric force is acting opposite to velocity of electron, so velocity of electron will decrease.

Magnetic force F '" q vB ... (i)

71.

72.

Centripetal force

mv2 F=r

... (ii)

From Eqs. (i) and (il), mv2

-- =qvB r

mv ,-"'-

qB

The time taken by the particle to complete one revolution,

T == 2nr v

2n mv vqB 211m qB

73. The internal resistance of the cell

r=(~11~12JR

== 240 - 120 2", 2(""'\

120 X ~L

V2 Rhot "'p

200 x 200 ==--WO--:-

=400(1 400

RC'old =10

",40(1

75. Magnetic needle is placed in non-uniform magnetic field. It experiences force and torque both due to unequal forces acting on poles.

76.

[Cr(NH3)4CI2t

Let oxidation state ofCr == x NH3 eo 0 Cl "'- 1

Net charge == + 1

[ Cr(NH3 )4CI2]+ x+4 x 0 +2(-1)=+1

x=3

Antipyretic drugs reduce fever. Analgesic releives in pain, antibiotics act against bacterial infections while tranquillsers are used against mental disorders.

78. A species is amphoteric if it is soluble in acid (behaves as a base) as well as in base (behaves as an acid).

Sn02 + 4HCl ~ SnCl4 + 2HzO

Basic Acid

77.

Sn02 + 2N aOH ---). N a 2SnO 3 + H20

Add Base

79. A species is said to be diamagnetic if it has all electrons paired.

MO Magnetic
Species Electrons Electronic
configuration behaviour
Hz- 3 ols" 0'1;.1 Paramagnetic
.Hi 1 ali Paramagnetic
Hz 2 01s2 Diamagnetic
He2' 3 o-ls20'lsl Paramagnetic 80. Na2S04 ~ 2Na·j. ,:. S'O~-

van't Hoff factor i '" [1 + (y - 1)0:]

where y is the number of ions from one mole solute, (in this case :« 3 ), 0: the degree of

dissociation. i = (1 + 2a)

81. Nylon-66 is a polyamide of hexamethylene diamine (CH2MNH2h and ' adipic acid (CH2)4 (COOH)2'

(Each reactant has six-carbon chain hence trade code (66) is used.)

82. Free radicals have unpaired electrons but are neutrals and are reactive.

83.

. .

CH3 + CH3 ---'Jo CH3 - CH3

t:.G'" '" - 2.303RT log Keq

I'!.GO = - nFE~ell'

If a cell reaction is spontaneous (proceeding in forward side), it means

Keq > 1 andE; e u =+ ve Thus I'!.GO eo - ve

4{ H -)0 iHe+ 2 ?e+ energy

It is the principal reaction of

Hydrogen-bomb.

85. Unit cell consists of A ions at the comers.

Thus number of ions of the type

A=~=l

84,

Unit cell consists of B ions at the centre of the six faces.

Thus number of ions of the type 6

B="2=3

[Each comers is shared by 8 cubes and each

face is shared.by 2 faces] .,

Thus formula is AB3•

86. Fluoro group causes negative inductive effect . increasing ionisation, thus 0.1 M difluoroacetic acid has highest electrical conductivity.

H 0

I II

F--+- C -+-C·-+-O--+-H

t

87. Greater the charge, smaller the radius, greater the polarising power and thus greater the covalent nature. This leads to increase in lattice energy.

88. X ....... Y is an endothermic reaction Ml =0 + ve

89.

E b == energy of activation of backward reaction E f == energy of activation of forward reaction llH;o heat of reaction

Thus. s, = Eb + M1

Thus Ef > s,

A13+ + 3e~ ~ Al

w==zQ

where w = amount of metal .

= 5.12 kg= 5.,12 x H?.3g z ~ electrochemical equivalent

EquIvalent weight Atomic mass

'" 96500 - Electrons x 96500

·27

3 x 96500

3 27

S.12x 10 = 3 x 96500 xQ

Q "" 5.12 x 103 x 3 x 96500 C 27

= 5,49 x 107 C

90. Size of colloidal particles = 1 to 100 run (say 10 - nm)

, 4 3 4 3

Vc == "3 nr ="3 11'(10)

Size of true solution particles ~ 1 nm

Vs =} 11'0)3

Thus Vc = 103 Vs

,91. Ml = IlU + 6ngRT

Ml = enthalpy change (at constant pressure) flU = internal energy change (at constant volume) (given reaction is exothermic) 6ng = mole of (gaseous products - gaseous rea ctants)

=-ve Thus ,!ill < 6U.

Note: Numerical value 01 t.H < tlU in exothermic reaction and when Ang < 0

92. MX2 . M2+ + 2X -

$ 2,

K~p == [M2+] [X-]2 If solubility be 5 then

K 'P == (s)(2 d ::= 4s3

4s3 == 4 X 10-12

s=l xl0-~ M M2+ == S = 1 X 10-4 M

93. Mixture contains 78g benzene = 1 mole benzene and 46 g toluene == 0.5 mole toluene

Total mole of benzene and toluene = 1.S mol

Mole fraction of benzene in mixture =: 1~ 5 = ~

VP of benzene p; = 75 torr

:. partial vapour pressure of benzene", p; Xb

= 75x~= 50 torr

94. Distribution of molecules (N) with velocity (u) at two temperatures TJ andT2 (1'2 >T1) is

shown below: '

At both temperatures, distribution of molecules with increase in velocity first increases, reaches a maximum value and then decreases.

95. 2N02,Cg) 2NO(g}+ 02(g)

tc, =: 1.8 x 10-6 at 184°C (::= 457 K)

R", 0.00831 kJ mol'" K-J

'where,

~ng = (gaseous products - gaseous reactants) = 3- 2= 1

.. K p = 1.8 x 10-6 x 0.00831 x 457

= 6.836 x 10-6> 1.8 x 10-6 Thus Kp > K,

96. Reaction is exothermic. By Le-Chatelier principle, a reaction is spontaneous in forward side (in the direction of formation of more ClF3) if F2 is added, temperature is lowered and ClF3 is removed.

97. pH= 5.4

[W 1 == 10-5.4 '" 10-6 ·10°·6 Antilog-of 0.6 is '" 4

.. [H+]", 4x 10-6 M

98. There are two different reactants (say A and 8).

A + B -----)0 product Thus it is a bimolecular reaction.

If :;; = k [A][8]

it is second-order reaction

If (:)=k(A]

or == k [8]

it is first order reaction.

Molecularity is independent of rate, but is the sum of the reacting substances thus it cannot be unimolecular reaction.

Total molarity == M 1 Vi + M 2 V2 Vi + V2

1.5 x 480 + 1.2 x 520 480+ 520'

== 1.344 M

100. During electrolysis, noble metals (inert metals) like Ag, Au and Pt are not affected and separate 'as anode mud from the impure anode.

99.

101. A~cOH = A~<.DNa + AHC1 - A~aCI = 91.0+ 426.2- 126.5 -= 390.7

102. Remains unchanged.

103. CA) Is (B) 2s

eC) 2p CD) 3d

(E) 3d '

In the absence of any field, 3 d in CD) and (E) will be of equal energy.

104. As we go down in the group, ionic character increases hence, melting point of halides should increase but NaCl has the highest melting point (800GC) due to its high lattice energy.

105. Variation of K.q with temperature T is given by _ van't Hoff equation.

su- Ll.SQ

log Ktq = - 2;303 RT + R

A B

Slope of the given line is + ve indicating that term A is positive thus Ll.H0 is - ve.

Thus reaction is exothermic.

106. Following reaction takes place during bessemerisation :

2Cup + Cu2S_ 6Cu + S02

107.

Moleeule Structure Hybridisation Lone
ofcentraJ pair
atom
F
], sp3d one
SF4 F~S~F
I
F
F
I sp3
CF4 F~C-F
I zero
F
F
XeF4 I sp3d2 two
F~:Xe:-F
I
F 108. Mixing the soles together can cause coagulation since the charges are neutralised.

109. Hypophosphorus acid (HJP02) is a monobasic acid and has only one ionisable H, two H-atoms are directly attached' to phosphotus. Thus the correct statement is (c),

H

I

H-P-O-H

II

°

110. Conjugate base is formed by loss of H+ .

OH - -----)0 02- + H+ Conjugate bru;e

02- is the conjugate base of Ol-l",

Ill. Aqueous solution of AlCl3 is acidic due to 120. hydrolysis.

AICI] + 3H20 ~ AI(OHh + 3HCl On strongly heating AI(OHh is converted into

AI20:! i .

2Al(OH)3 ~ AIZ03 + 3HP

112. In a group, lJ.G f 0 (HX) changes from. - ve to + ve downwards. HF(g)lJ.G ~- 273 .20 kJ rnol'" HI (g) lJ.G = + 1.72 kJ mol"!

Thus HF is, thermally stable and H1 not. Thus HF > HCI > HBr > HI

113. Hg2Clz + 2NHJ ~ HgNH2Cl + Hg + NH4Cl

white black

114. CaCz (Calcium carbide) is ionic.

CaCz~ Caz++C~-

-c= c-

C ~- has one (J and two rt bonds.

115. Crp~- + 14H+ + sr ~ 2Cr3+ + mp + 312 Crz0;- is reduced to Cr3+ .

Thus final state of Cr is + 3. Hence, (a)

117. Lanthanide contraction, cancels almost exactly the normal size increase on descending a group of transition elements, thus Nb and Ta and, Zr and Hf have same covalent and ionic radii.

118. K3 [Fe{CN)('l

Cation Anion

Oxidation state of Fe in anion « + 3

Thus it is potassium hexacyanoferrate (III).

119. (a) Metallic radii increase in a group from tOP to bottom.

Thus Li < Na < K < Rb -True

(b) Electron gain of enthalpy of Cl > F and decreases along a group

Thus I < Br < F -c Cl is true.

ee) Ionisation enthalpy increases along a period left to right but due to presence of half-filled orbitals in N, ionisati.on enthalpy ofN > 0.

Thus B < C <. N < 0 is incorrect.

Spectes Electron Electronlil ,Charge Total,
in central in other gained
element element
BOt S 3 x 8 = 24 +3 32
CO~- 6 3 x 8 = 24 + 2 3,2
N03 7 3 x 8 == 24 + I 32
SO~- 16 3 x 8 = 24 +2 42
CN- 6 7 1 14
N2 7 7 0 14
Cr- 6 6~ +2 14
PO~- IS 4 x 8", 32 +3 50
SO~ 16 4x 8 '" 32 + 2 50
ClO4 17 4 x 8 == 32 +1 5Q Thus (a) BO~-, CO~-, NOj" are isoelectronlc, (b) SO~-, coi-, NO:! are not isoelectronic,

121.

3° (C-H) bond has

Iminimum bond energy

H H hence easily cleaved giving

! ! 2 bromo 2-methyl butane

H3C-,-r-CH3

H CH3

Br

1Ir2 - I

--l-CH3-CHz -C-CH3

I

eH3

122,.

3-

123.

Mirror image is not superimposable hence, optical isomerism is possible.

Hybridi. Unpaired M~gnetic sation electrons moment

(a) [CO(CN)6]3- d2sp3 0 0
(b) [F~(CN)6]J- d2sp3 . 1 i3BM
(c) [Mn(CN)6]3- dZsp3 2 .J8BM
Cd) [Cr(CN)6]3-- d2sp3 3 mBM
rhus least paramagnetism is in Cal. 2'

124. CH3 -CH-CH-CH-CHJ ~

I ' -

OH

CH3 -C-CH =CH-CH3

II o

Only sititable reagent is chromic anhydride in glacial acetic acid. Other will also effect (C =C)bond.

125. riMg + y ------------')0 ~H + ;;Na

Thus nuclide of UNa is formed. ~26. CHz = CHCH = CHz + HBr-----)

CH3 CH CH = CH2 + CHaCH = CHCH2Br

I

Sr

I ,2·addition product 1, 4·addition product

Addition is through the formation ofal1ylic carbocation

® ~

CH2 =CHCHCHa ~ CH3CH= CHCH2

(2' a \lylic) (1" aUylic)

(more stable) (less stable)

Under mild conditions (temperature "" - 80o,C) kinetic product is the 1. 2-addition product and under vigorous conditions (temp. "'40Q C) thennodynamic product is the 1, 4-addition product.

Thus l-bromo-z-butenc is the major product

under given condition. -

127. If acid is weak, its conjugate base (nudeophile) is Strong and vice versa.

(A) - 0

II

CH3 -C- 0- is a conjugate base of

o

II CH3COH (1)

. ' ,

Ca,) CHP- is a conjugate base ofCHPH (II)

{C) 'eN - is a conjugate base of HeN CUI)

(D) H3C ---O---SO~ isa conjugate base of H3C ---O---S03H(IV)

. Acidic nature of IV > I > III > II and nucleophilicity eifB> C:> A:> 0

128. In.~ 2 reaction, nucleophile and alkyl halide react in one step.

1 . R", I

R -I-Br+ Nu" ----) Nu C Br

R 1

R

Thus tertiary carbon is under steric hindrance thus reaction does not take place until (C-Br) bond breaks

R R

I ' ' I

R -C-Br ~ R~Cjj) + Br"

I I

R R

which is then SN 1 reaction.

129. Synthesis of RNNDNA from phosphoric acid, ribose and cytosine is given below

Thus ester linkages are at C;' and C1 • ofsugar

molecule. _

'. '~eh.Y:ratio\~:2'l

, _?~ I HOCH~ 0 OH ~ 0

O-p OH 4~1 .H ________

6H H .•. ' H2.· -<'H20

OH H

OH .A..

r 5' sl.l,3l

o=r-O-CkH-0~~ 0

OH 4~A;'

OH OH

Out of the given acids, strongest is HeOOH. (highestKa value)

Since pKa", -.log Ka

Thus lowest pKa is of HCOOH . el2

(A) 2-methyl pentane ~ five types of

monochlorinated compounds

- '.. .' O2

(8) 2, 2~dimet_hylbutane ~ three types .

(C) 2, 3"dimethylbutane ~ two types .

Clz

(D) a-hexane ~ three types .. " .

132. It is Corey House synthesis of alkanes.

133. Wurtz reaction is used to prepare alkanes from alkyl halides!

130.

131.

2R -X + 2Na d.ryelher R -R + 2NaX

134. CHa -CH-CH-CH3

I )

CI C!

There are two chiral C,atoms (*) Thus optical isomerism is possible.

135. (a) ~

N02 o

II

(b) CHJCNH2

o

II

CH3C group is also electron withdrawin~.

N02 group (electron with

-drawing) decreases basic nature of anllirie,

(c) @-NH2 phenyl group is also electron withdrawing.

(d) Benzyl is electron-repelling group increases

basic nature. . .

Thus most basic compound is benzylarnine.

136.

(JI

(2° alcohol) through 2° carbocation CH3CHCH3

CHa -C = CH2 --~ (CHlhCOH

I

; ID

(3· alcohol) through 3° carbocation (CH3)3C

CH3

I

CHJ -CU- CH = CH2

. CH3

H.O/H+ . I

-2~1 C!'I:.C~:rbo!o-:CH3

1,2 tr shift

1'120 2° alcohol

CH3

I H20

CH3-C-CH2-CH3 ---+ 3" alcohol $

JDcnrbocation .

Thus best alternate is ec)

137. TeHon is -f-CF2-CF2+n

138. CI·l3CH2CHCH3 ~ CH3CH2CHCH3 ~

. I Ill-H'

fir

CH3CH2CH=CH2 + CH3CH=CHCH3

Less substituted More substituted

II I

Stability of I > II hence I is predominant.

139. Boiling point and freezing point depend on Kb (molal elevation constant) and Kf (molal depression constant) of the solvent. Thus equimolar solution (ofthe non-electrolyte) will have same boiling point and also same freezing point.

ATj = K / x molality ATb == Kb x molality

Note: In question (139) set C, nature of solute has not been mentioned. Hence, we have assumed that solute is non-electrolyte.

o 0

II II

140. R-C-X + Nu--.~ R-C-Nu + ;t-

Best leaving group (poorest nucleophile) isCl0, thus fastest reaction is with CL

141. B2H6 has structure

Terminal boo<ls (2C-2el

Bri<lglng bonds (3C-2e-)

HJ

o

•• • Terminal

.60 B. bond_

\ H.\ a 2C-2a

~H

• Boron electron

o rJydrogen electron

Bridgin.9 bonds (3C-2e-)

142. Hydrogen atom is in lSi and' these 35, 3p and 3d orbitals will have same energy w.r.t. ls orbital.

143. Lanthanide contraction is due to poor shielding of one of 4f electron by another in rbe sub-shell.

144. (a) dS in strong field

n == unpaired electron = 1

-----<Feg nl1lt2;g

Magnetic moment ",.J$ + 2)BM

= -J3 8M = 1.73 13M

(b) d3 in strong/weak field ---<fiOg

~UU11t2g

n=3

Magnetic moment == J1s = 3.87 BM (e) d4 in weak field

/IDeg -----<WI1 I t2g

11=4

Magnetic moment =m = 4.90 BM (d) d4 in strong field

fieg ----<WI 1 I t2g

n=2

Magnetic moment == Fa = 2.83 BM

145.

C>= <CH3 c>- <CHs

O+HN -+ N

CH3 - CH3

.: 1l enamine

C)-e<CH~

Enol ' CH3

OH

146. $ + CHd, + orr CH3

p-cresol

Reimer-Tiemann reaction

OB b-OH

HCN ) I

CN

CBs'

OH II

H 0+ c$r!~OH

3 ) 0 I

COOH

CH:~

147.

(-QH is more activating than -CHJ in 0, p-directing, thus -CHO goes to ortho w.r.t -OH)

Formation of XY is shown as X 2 + Y2 ______. 2XY

l!.H = (BElx_x + (BE)y_y - 2(BEh_Y If (BE) of X - Y = a

then (BE) of (X - X) == a

and (BE) of (Y - Y) "" ~

. • t:.H f eX - Y) == - 200 kJ

a

.. - 400 (for 2 molXY) == a+"2 - 2n

-400=-~

2

a=+ 800 kJ

The bond dissociation energy of

X 2 = BOO kJ mol-1

NH4HS(s)<;==± NH3(g) + HzS(g)

148.

initially 1

at equilibrium Q - x)

0.5 0 (0.5 + x) x

Total pressure at equilibrium

= PNH3 + Ptl2S =0.5+ x+ x=D.B4

x = 0.17 atm

PNH3 = 0.50+ 0.17= 0.67 atm

PH2S = 0.17 atrn .

Kp = PNH3 . PH2S

== 0.67x 0.17= 0.114 atm

149.

Element Percentage Percentage Simple
at. wt. Ratio
C 20.0 2~20 = 1.66 1.66 _ 1
1.66 -
H 6.67 6.67 = 6.67 6.67 " 4
1 1.66
N 46.67 46.67 = 3 33 3.33" 2
14 . 1.66
0 26.66 2~:6 = 1.66 L66 =1
1.66 Empirical formula « CH4NP Empirical formula weight

'" 12+ (4 x 1)+ (2.x 14)+ 16 =60

~ '" Mol. formula weight = 60 = 1

. . Emp. formula weight 60

.. Molecular formula =: CH4N20

Given compound gives biuret test. Thus given compound is urea (NH2)2CO.

...

NH2CONH2 + HNHCONH2 ~

. i CUS04

NH~ONHCONH2 + NH3 ~ Violet colour biuret

150.

.' A ~ Product
initially a 0
after time t (a - x) x
after tl/4 (a -~) a
4 For first-order kinetics,

k _ 2.303 I ( a ) --- og ~.-

t a-x

k = 2.303 log ~

tl/4 . 3a

4 4

. 2.30310g"3

tl/4 = k

0.29 r=«:

_,.......___-----

1. Key Idea: If C is mid point of AB andP bethe

---) ---)

.. h P---)C PA + PH

orlgm, t en = 2

Let P be the origin outside of AB and C is mid point of AB, then

---) ---) ---)

2PC= PA + PH

2. The co-ordinates of Pare (1, 0). A general point Q on y2"", 8x is (2t2, 4t). Mid point of PQ is

(h, k), so

2h=2t2+1 .•. (i)

and 2k"'4t=> t=k/2 ... (ii)

On putting the value of t from Eq. (ii) inEq. (0,

we get

2k2 2h==~+1 4

=> 4h'" k2 + 2

So the locus of (h, k) is y2 - 4x + 2 = O.

3. Key Idea: The relation among Mean, Mode and Median of any frequency distribution is Mode", 3Median - 2 Mean.

Given that Mean » 21 and Median == 22 Mode = 3 Median- 2 Mean Mode = 3 (22) - 2 (21)

== 66 - 42

=24

4. Since, (3, 3), (6, 6), (9, 9), (12, 12) eo R => R is reflexive relation.

Now (6, 12) eo R but (12, 6) i'" R, so it is not a symmetric relation.

Also (3, 6), (6, ]2) E R => (3, 12) E R

=> R is transitive relation.

Note : Any relation is said to be an equivalence relation, if it is reflexive, symmetric and transitive relation simultaneously.

5. Key Idea: If A is any square matrix, then AA-1 =IandA-1I=A-1,

Since, A2 - A + 1=0 ..

=> A-1A2_A-1A+A-1I=0

(Pre multiply given relation by A-I)

=> (A-1A)A-(A-1A)+K1=O

=> A-'I+Kl=O

:=} KI ""I - A

6. Since (x -ll + 8 = 0

=> (x -IP '" - 8=(_2)3

=> (x_-21r "",1

=> (X:21) ""'(ljl!3

., roots of ( x _-.}) are 1, (I) and (02.

:=} roots of ex - 1) are -2,- 2m and - 2m2

=> roots roots ofx are -1,1- 2m and 1 - 2m2• Note: If 1, ro and ro 2 are cu be roots of un lty, then

1 + ro + ro2 "" 0 and ~)3 '" 1

(1 21 2 24

7. Let A '" lim - sec - + - sec -

n->'" n2 n2 n2 n2

1 2)

+ .. ,+ - sec 1

. n

'1 (1 (1)2 2 (2)2

lim - - sec2 - + - sec2 -

"_,,., n n n n . n

( )2)

11 . -2' n .-

+ +;sec Ii:.'

_ 1 rj lim - L

rl-)-:t:I n r .. 1

I

(~) 5ec2 (fir

Let

A '" f~ x ~ec"· (Xl) dx Xl =£

2x dx = dt ,::), xdx = dt ,.' 2

A = _! fl scc2 t dt 2 {)

1 Jl 1

"" '2 [tan t . 0 =- 2. tan 1

8. Key Idea: The pamrnetl"ic co-ordinates of a

. xl y2

point that lies on an ellIpse -f + 2::.1 are a b

(aC050, b sin 0).

Let the co-ordinates of the vertices of rectangle

ABeD are II (0 cos 0, b sin 0),

B (- a cos 0, b sin 0), C (- a cos 0, - b sin 0)

and D (n cos 0, - b sin 0), then

length of rectangle, A/3 = 2a cos 0

and breadth of rectangle, 110 = 2b sin 0

Area of rectangle", AB x AD

-= 2a cos e x 2.b sin ()

==> Area of rectangle, A '" 2 ab sin 20 ... (0

y

{- a COSO. - b sinO)

y'

dA

dO = 2 x 2. ab cos 20

On putting ~ = 0 for maxima or minima.

.. ~=o

c0520=0

Now

(d2A), <0

do21)~':: 4

A· . 0 1t

.', rea IS maximum at . = "4 .

Now

==> Maximum Area of rectangle = 2 ab sq unit. (From (i))

Alternate Solution From Eq, (i)

Area of rectangle, A "" 2 ab sin 2G .; A r:.( sin 20 and- 1 !O sin 20::>, 1 ., A is maximum when sin 20 ee 1

==> Maximum area of rectangle» 2absq unit,

9. Key Idea: The differential equation of ajamily of curves of II. parameters is a differential equation of II. maximum order.

Equation of family of curves is

y2",,2c(x+..JC) ... (i)

On differentiating Eq, 0) with respect to x, then 2JYl = 2c

C=)')'I

On putting the value of c in Eq. (0, we get y2 "" 2.»'1 ex + /Yi;)

10.

.. The degree and order of above equation are 3 and 1 respectively.

Key Idea: If three forres acting 0.0. Q. particle keep it in equilibrium, each is proportional to the sine of the angle between the other two.

p Q R

sin u - sin ~ ;; sin y

This theorem is known as Lami's theorem.

R

In a t.. ABC, I is the inccnrrc, _. A

. , ,

'~~

. , ,

)-~ .

Q __ ~---_ R

/' - .... -

• /6/2 -

B

c

LBIC ;= rr - (% + ~) =n-(1-4)

1t 'A =2+2"

Similarly, L AlC = % + % and LAIB = ~ + %

By Lami's theorem

_1__ := _1_ == _!__

sin BIC sin Ale sin ALB

:.1:0:R

. (7t A) . (7t B) . (n C)

=0 Sin "2 + 2" ; Sin "2 +"2 : Sin 2 + 2:

ABC sCOS"2:COSz-:cosz-

11. Key Idea: (1) The coejficient oj (r + 1 Jtll term oj(l + y)'" is mer-

(2) if a, b, c are in AI', then b '" II; c.

Since, me r:'1 + mC r .. 1 == 2 "C r

m! m!

~ . + ~----~~

(r-l)!(m - r + 1)1 (r+ l)!(m - 1'-1)1

m! . ==2 r!(m-r)!

1 1 2

;;:) . + --- == --,.----,.

(m- r+ IJ(m - r) (r+ 1)r rem - r)

r(r+ 1)+ (m - r + l)(rh - r) 2

;;:) . '" __,.---c

r(r+ 1) (111- r+ l)(m - r) rem - r)

;;:) rZ + r + m2 + ,.2 - 2mI' 4: 111 - r

= 2(mr - ,.'2 + r + m - r + 1)

;;:) 41'2 - 4mr - m - 2 + m2 = 0

;;:) 1Il2-m(4r+l)+4r2-2=0

12. Key Idea : If a and ,I are the roots of the ~quatiol1 ax2 + bx + c == 0, rhen a + fl == -b and a

c

all '" -'.

a

Since, tan (i) and tan (%) are

equation ax2 + bx + c == ° .

P . Q b

tan z- + tan "2 == - a

P Q c

tan z- tan "2 Ooa:

f..+g_+~==~ 2 2 2 2

roots of

and

Also,

(As P, Q, R are angles of a triangle) P + Q 1t R P +'Q 1t -2-.="2-Z- ::::> -2-==4

Now, tan (%+ i) = 1

P Q

tan -+ tan-

2 2 = 1

P Q

1- tan "2 tan 2"

b

-(1 b c

--=1=>--=1--

1-!:. a a a

=> -b",a-c::)c=a+b

Alternate Solution

LR=~

2

LP+ LQ ==~ 2

LP 7t LQ 2="4-2

tan( %) == tan (~- ~)

tan i - tan%

1 + tan ~ tan ~ tan ~ + tan E: tan Q = 1 - tan 9.

2 2 2 2

tan .£ + tan g_ = 1 ..: tan ~ tan 9.

2 2 2 2

b c

--==I--::::>-b=a-c

a a

~ c= a+ b

13. In SACHlN order of alphabets is A,C,H,I,N,S.

Number of words start with A == 51, so with C, H, 1, N, Now words start with .S, and after that ACHIN are in ascending orders of position so 5· 5 ! == 600 words are in dictionary before words with S start and position of this word is 601. '

14. Key Idea: "C, + "Cr + 1 = .. IC,. tNow SOC + SSe + 54C + 53C + 52 ....

, • 4 :I 3 3 l..3

+ SIC3 + soc]

= 50e3 t 50C4 + SIC:) + S2C3 + 53C3

+ 54C3 + 55C3 = SiC,! -+ SIC3 + SZC3 ~ 53C3 + 54C3 + SSC3

(": 't, + "C'-l '" n+ Ie,) = 52C4 + 52C3 + 53C3 + 54C3 + S5C3

= 53C4 + 5.3C3 + 54C3 + SSC3

== 54C4 + 54e 3 + SSC,J

_ SSe + SSe _ 56C

-1 ' 3~ 4

15. A=[1 ~J

AZ =[i ~JU ~J A:=[~ ~J A3=[~ ~][i ~l

=[~ ~]

An=: [~ ~] can be veri fled by ind uetion. Now, go option by option

(b)[; ~J;O[~ ~J+[n~l n~iJ

(d)nA-(n-l)[=[~ ~J_[n~l n~1]

= [~ ~J .

16. Key Idea: The (1' + 1) th term in the expansion af(x + at is nCr xn-r ar•

Let x7 is contained in (1' + l)th term in the

'expansion of ( axZ + txy1.

T = llC (ax 2)11 - r (_.!_) r

.. r+l r bx

11 - r

= IlC _a _ ,X22 - 3,

r b' .

::;) 22 - 3r = 7

::;) 3r = IS

:::::> 'r= 5

11 a6 7

.. T6 =, C 5 bS • x

., Coefficient of x7 in the expansion of

( 2 1 )11 11 a6 ox + bx = C5 bS

Similarly, coefficient of x-7 in the expansion qf

(aX __ 1_) II = "c 6 as bX2 b6

According to question,

6 5

He ~_llC s.

5 bS - 6 b6

a6 as

bS = b6

ab= 1

17.

Since x E (-1,1)

-1 (1t 1C)

tan X E - 4' '4,

2 -1 (It 7t)

tan x E -2' 2

and f(x) == tan-1 ~ = ztan ? x, I-x

So, feX)E(-%.~)

:. Function is one-one onto,

(X2 < 1).

=>

18_ Let Z"1 = Xl + tyl and Z2 = x2 + (yz Now given that

1 ZI + z21 = 1 211 + 1 z21

=> ~(XI + x2i + (Yl + Y2i

= ~ xi + Y? + ~ x~ + Y~

2 2 2 2 2 2

=> Xl + x2 + XIX2 + Y1 + Y2 + ~IY2

2 2 2 2 2 ~( Z 2) (2 2)

= Xl + YI + X2 + Y2 + XI + Yl Xl + Y2

:::::> (X1x2 + YIY2) == ~(Xi + yt) (x~ + Y~)

::;) xtx~ + Y~Y~ + 2x1X2YIY2.

22 22 22 22

= X1X2 + YIY2 + X1Yz + YIX2

XIYl - YlX2 '" 0

Y2 '" Yl

X2 Xl

tan " (~:) = tan"! (~:)

:::::>

=>

=> arg Z2 '" arg ZI

=> arg z2 - arg z1 '" 0

Alternate Solution

.: I ZI + z21 = I ZI 1 + I zzl On squaring, we get

I ZI + z212 '" (I zd + ! Z21)2

2 2 -

=:> 121 I + I Z21 + 2Re (ZI2Z)

=12112 + Izi + 21 zIIIz21

=> Re (z} 22)=1 Zl II z21

::;) I ZJc II 221 cos (~ - 8z) == I zlll z21

=> f\-02=0

=> arg (21) - arg (22) = 0

19. Key Ide~ : If I 2 - ,2, 1,,1 = k, then

z - 22

z lies on. a circle of k o!; I and z lies on a perpendicular bisector of segment with 'end points ZI and 22 , -if k = 1.

Given that w = ~ and I wi = 1 2-3 .

=> -;- =1 ~ Izl=!z-~1

z-'3

=> z lies on .L bisector of (0, 0) and (0, ~) .. So, z lies on a.straight line.

Altema.te Solution

z w=~andlwl =1 l

z-'3

=>

z =1

i

z-3

=> 3121=132-i)

Letz=x+iy

.. 31 x + iy I = 13 (x + iy) - il

=> 3 Jex2 + y2) =::Jc3xi ... (3y -Ii

=>

:. Which shows that z lies on a straighr line. 1 + (12 X (I + b2) x (1 + c2 J x 20. fex)= (1+chx l+bz x (l+c2)x

(I + al) x (I + b2 J x 1 + ,2 X

ApplY'jog e1 ~ C1 + C2 + C)

1 + aZ x + x +"b2 X + X + e2 x

f(x) = x + aZ x + 1 + b2 X + X + ,2 X X + a2 x + x + b2 X + 1 + c2 X

(1 + b2) x 1 + b2 x (1 + b2)x

(l + e2) x (1 + 'e2) X 1 + e2 x

. 1 + Ca2 + b2 + eZ + 2) x = 1 + (a2 + b2 + ~2 + 2) x 1 + (a2 + b2 + c2 + 2)x

O+b2)x 1 + b2 x (1 + b2)x

(1 + e2)x (1 + ~)x 1 + c2x

1 (l + b2)x (1 + ,2) x
=.1 1 + b2 X (1 + ~) x
I (1 + b2)x 1 + e2 x R} ~ R} -R3, Rz~' Rz -RJ

o 0 x-I

=0 1- x x-I

I (l+bz)x I+c~,x

lox-II

= I-x x-I

'" (x _1)2

=> fex) is of degree 2.

21. The system ofgiven equation has no solution, if (1.. 1 1

1 1

=> I~:;

(1..+2

(Cl ~ C1 + ez + el)

=:> (0.+2) ~ (1..~lb'I·=o

o ° (1..-1

(Rz ~ Rz -RI, RJ ~ R) -RI)

=> (a + 2)(0: - Ii = 0

=> a =1, - 2

But 0: '" 1 makes above three equations same. So, the system. of equation have infinite solution. So answer is a "" - 2 for which the system of equations has no solution.

22. Let u and P be the roots of equation' XZ - (n - 2) x - a-I = 0, then a + fl = a - 2 and ap, = - a-I 'Now, 0:2 +p2 '" (a +'p,p - 2aP

=> 0.2 +p2 ",(a-2)2 + 2(a+ 1).

=> (X2 +p2 =a2 + 4-4a+ .20+ ~

=> II 2. + p2 = 02 - 20 + 6

=> o:Z +p2"'(a_'1)2+ 5

The value of 0:2 + p2 will be least, if a-I = O.

=:) a=l

Alternate Solution

a + J3 = (a - 2) and uJ3= - a-I

fea) = ~2 + pZ::: (0: + pi - .2a.j3 '" (a - 2)z + 2(~+ I) ;"a2-2a+6

=:> f' (a) = 2a - 2

On putting f' (a) = 0 for maxima or minima.

a 1 ",0
1 a:
1 1
a 1 =0
1 a 2a-2=O a=l

23.

Now I" (a) == 2

~ f" (1)", 2> 0

:. f(a) is minimum at a == 1 -. ,

Key Ideoi: If cheroots of ['he equation x2 + bx + c'"' 0 ere consecuave integers, chen

_the value ofb2 - 4c is always J,

Letn and en + 1) be the roots of x2 - bx + c'" 0 then n + (n + 1) = band n(n + 1) == c

.. b2 - 4c =,(2n + Ii - 4n(n + 1)

= 4n2 + 4n + 1 - 4n2 - 4n

=1
24. f'(1)= lim /(1 + h)- /(1)
/,-.0 --h-------
lim /(1 + h) - lim /(1)
h->O It '1 ........ 0 T Now, lim /(1 h+ h) == 5, so lim fh(1) must be

/,->0 11-.0

finite as i' (1) exist and lim /(h1) can be finite

/, .... 0

only, if f(l) == 0 and lim f/(1) = o.

/, -00 t

So /'(1)= lim f(1+11)=5.

10-.0 h .

25. Given that, f(1) == - 2 and 1 '(x) ~ 2 dy

Tx~2

~ dy;:;: 2dx

==:> J.J(6) dy ;:;: 2 r dx

~(lJ 1

::;::) /(6) - f(l) ~ 10

=> /(6) ~ io -+- f(l)

=> /(6);:;: 8

Alternate Solution

1(6) - f(1) > 2 6-1 ~

::;::) f(6) - f(l);:" 10

=> f(6) + 2;:., 10

=> . f(6) ~ 8

Jim I/(x)-fCyJI ~ lim Ix-YI

¥---'oJ Ix-YI x->y

~ If'Cy)I::;O

~ I'Cy)== 0

::;::)- fCy) = constant

as f(O) = 0

=> f(y) = 0

=> f(1) == 0

26.

27.

(1 + X)1I2 -( 1 +~x r (1- xi!2

{(I + ~X + ~(~)G) X2+ ••. ) .

- ( 1 + ~ x + ~ x2 + .. .)}

(1 - xi!2 = _ 3x2 (1 _ xrl/2

8

=- 3f (1 + ~x+ ~(~) G) x2 + .. J

3x-2

= - 8" + higher powers of x.

3x'2

-8"

C': higher powers of x greater than x2 can be neglected). .

28. .: x = fa"'''' 1 + a + a2 + ...

n=O

1

~ x"'l_a

Similarly,

1 1

Y =-- and 2 =,--

1-b 1-c

Now, a, b, c are in AP - .

=> - a, -, b, - c are also in AP.

~ 1 - a, 1 - b, 1 - c are also in AP.

1 1 1 . HP

=:) 1_a'1_b'l_carem .

=> x, y, z are in HP.

Alternate Solution

1 1 1

x~l-a,y==l-b' z==l_c

\x-1 _ y-l 2-1

~ a==.-- b=-- c=--

. x' y' 2

.: a, b, c are in AP

2b = a+ c

(y-1) x-I z-I 2 -- =--+--

Y x 2-

2 1 I

2--=1--+1--

Y x z

211

- =- +,_

Y x z

::;::)-

=> x, y, z are in HP.

c '

29. We know that ~C = 2R

, sin

c= 2R

... (i) (": C == 90")

and

C I" tan 2' "" s - c

n r tan -=-! 4 s - c

30.

I"'="s-c

a+ h+ c

~ r=--2---c

~ 21" '" a + b - c

On adding Eqs. (I) and (H) ~ 2 (r + R) = a + b

Key Idea: cos " x: ± COS""1 y

'" cos-1 (.xy +" ~1=;'(2 ~1 _ y2).

... (ii)

31.

We have", cos'" x - cos-1 ~ ::: a

~ '"'t~ +)i - x' ~1 - y: ) = • .'0' ~g "

~ - -1-;11 - X" ;; cos (X

2

~ 2)1 - x2 g '" 2 cos n - .xy 4(1 x2) (4 y2)

""" - - = 4 cos2 a + x2y2

4

- 4_W cos a =:4 - 4x2 - y2 -I- x2y2 == 4 cos2 U. + x2y2

" - 2.xy cos a

=> 4x2 - 4.xy cos Cl + y2 "" 4 sin2 Cl.

Since in 6 ABD

A

c

ALi '" C cos (900 - B) AD = C sin B Similarly,

BE = a sin C and CF '" b sin A . : AD, BE, CF arc in HP.

:. c sin B, a sin C, b sin A arc in BP.

1 1

1

sin C sin B' sin A sin C' sin B sin A'

are in AP.

~ sih A, sin B, sin C arc in A~.

Alternate Solution

1

or (A ABC)"" '2 xBC xAD

1 26

=> b"'2xaxAD~AD=a

Similarly, BE ::: 2: and CF =' 2:

.,' AD, BE, CF are in HP.

111·HP

:·u'b'carclO .

""" a, b, care InAP.

=> sin A, sin B, sin C are in AP.

32. Key Idea : Equation of normal at any point" (Xl> yd on any curve is

1

y - Yl = - (':)" (x - xd

(-"I.Y) ) Given that X = a (cos 0 + e sin 8)

~ == a (- sin 0 + sin e + e cos 0) dx

""" dO=aecos~

Y = a (sin 0 - 0 cos 0)

dy .

dO = a (cos e - cos 0 + f) sm 0)

dy =aesin f)

dO

ay dYldO

dX "" dx/ de = tan a

Slope of normal

=-: ",-cot 6= tan (~+e)

and

So equation of normal is y- asin 0+ a o cos 9=

cos e "

_'-.- (x _ a cos 0- aasin 0) Sin 0

~ sin Oy - a sinz a + ~ e cos a sin a

"" - x cos 0 + a cos2 a + a e sin e cos e

x cos (3+ y sin f:l=a

33 .

It is always at a constant distance 'a' from origin.

(a) f(x) '" x3 + 6x2 + 6

f' (x) '" 3x2 + 12x '" 3x (x + 4)

+.

+

-4 ,

a

t

X E(- 00, - 4) '"" (0,00)

. (b) f(x) = 3x2 - 2x +1

.. t

I t

..

1 + "3"

F(x), =; 6xL 2

34.

:::>. f i ~x) > 0 "It x ~ (~, co )

This is wrong.

I. 1 ..,. cos (ax2 + bx + e)

1m - ..... --'----=--~

x4u .. (x -al

2 ~ 2 (ax2 + bx + e)

s~n 2

= lim

(x - a)2

. 2siri2 (~ ex - «)(x -In} (a)2 2

= lim 2 - (x -Il)

x·->u (a) 2 2 2

2. ex ~ a) ex - P)

.ci 2a2 2

= lim - ex -j3} =-(<<-13)

X-HI 2 2

.. ' Sin2(~(x-a.)(X-{3») r? 2

.. lim . 2 = -2 ell. - 0)

x .... " (a .\

2. ex - a)(x - P») .

35.

Note : If a and ~ are the roots of the equation ax2 + ox + c = 0, then this equation can be rewritten.as a (x - a) (x - jl) '" O.

x * = y (log y - log x + 1)

ft = (f) (log(~) + 1)

Now put }'_ = t x

=>

dy dt

y '" t x => dx =t + x ax

at

t + x dx ::. t log t + r

t log t dx = x dt

dt· dx

t logt= x

log (~) = ex

=

36.

Key Idea : If £1 and L2 are two lines, then the equation of line passing through the point of intersection of 1.1 and L2 will be 1'1 + AL2 ::. o.

Equation of a line passing through the intersection of lines ax + 2by + 3b == ° and bx ~ 2ay - 3a ::. 0 is

(ax· + 2by + 3b) + h.(bx - 2ay - 311) = ° ... (i) Now, this line is parallel to x-axis, so coefficient

a ofx=O~ a+ Ab= 0::> ""=-1)'

On putting this value in Eq. (i), we get b(ar + 2by+ 3b) - a(bx - 2ay - 3a) = 0

=> 2b2y + 3b2 + 2a2y + 3a2 = °

=> 2(b2 + 02)y + 3(b2 + 02)= O.

3

y=-Z

Therefore the required line is below x-axis, at

di 3 f .

a istance Z 'rom It.

Alternate Solution

Equations of given lines are ax + 2by + 3b",0

and bX-2ay-3a=0

On solving Eqs. (i) and (ii), we get 3

x=O,y=-Z

.. , Point of intersection of lines is ( 0,

, .. (i) ... (ii)

Also required line is parallel to x-axis,

.. m=O

:. Equation of required line is

(y + ~) '" 0 (x - 0)

3 y=-2.

38.

Thus the required line is below x-axis at a

d· 3 f .

istance 2: rom x-axis,

Given that ~~ -= 50 em] I min

:c (;xr3)=50

dr3 150 di- = 4n-

31'2 dr = 150

dt 4x

dr 50 ·de == 41(r2

(dr) 50 1 .

=> - == -- cm/ mm.

dt r - rs 4n x 225 18 1C

LetI=- f ( logx-l )2dx

1 + (log xi

=J

(log x)2 + 1- 210g x ax (1 + (logxi)2

= f dx f 2logx ·dx

1 + (log xi (1 + (log xy)2 .

x J 2logx dx

== 1 + (log xi -I- (1 + (log xii

f 210gx ~

- (1 + (log x)2/

x

== +c

1 + (log xi

lim f/(X) 4(3 dr

x-+2 6 x...;. 2

(Xl4t3 dJ:

= lim _..:;..6 ~----=:---

x -+2 ex - 2)

= lim 4 {f{x)}3 f' (x)

x-+2 1

= 4 {f (2)}3 f I (2)

=4x(6i x_!_

48

=: 18

J:./(X) dx == ~ sin P + * cos(l + .J213

Differentiate with respect to p on both sides tOl)= sin 13 + P cosJ}- ~ sin p +.J2

(1t) rt - 1[ tz:

So f -. =1+0--+.,12=1--+'112

'2 4 4

r1 2' rl 3

41. II = Jo tK dx, 12 = '0'Z dx,

39.

40.

f12",,2 12..,..9

fa == L. dx and 14 = l':'dx

tK3 < z:2 0 < X < 1

ZC3 > 2?'2 X > 1

:. I2 < I1 and 14 > 13

. fO

42. Required <_lrea A == 1- < log. ex + e) dx

'X==-C ,

, y'

Put X + e :: t => dx = dt

. . A:: J; log. r dt = [t log t - r ]~

=: (e- e- 0+ 1)= i

43. Key Idea: Area of region bounded by y2 = 4ax

d 2 4b . 16ab .

an y == !y IS -3- sq urur.

Now,

y.

i'"'4y

1 64 16 .

=: 12 x = "'3 sq Unit

52 + 53 = I: -I4X ax

[ ]4

x3/2 4

=2x2 ----'3" o=3X8

32 .

= "'3 sq unit

5 16 .

2 :-sq umt 3

16 16 16

51 : S2 ; S3 = "3 : "3 : "3 ~ 1 : 1 : 1

44. Key Idea : Centre of" sphere

x2 + y2 + z2 + 2ux + 2ry + 2wz + d:: 0 is

(-u, -II, -w).

Equation of first sphere is

, x2 + y2 + Z2 + 6x - 8y - 2z = 13 .... 0)

whose 'centre is (-3, 4, 1)

and equation of second sphere is

x2 + y2 + ~ -lOx + 4y - 2z = 8 ... (il)

whose centre is (5, -2, 1).

Mid point of (-3, 4, 1) and (5, - 2,1) is (l,.}, 1). Since, the plane passes through (1, 1, 1).

2a-3U+ 4a+ 6=0 .

~ 3a=-6 =:) a=-2

45. Line is parallel to plane as

(i - ) + 4' ). ci + 5) + , ) '" 0 .

General point on the line is {A + 2, ._ ,}, - 2, 4t.. + 3) . For '}., = 0 a point on this tine is (2, - 2, 3) and distance from

_, - • .r_

r ',(i + 5j + K ) == 5 or x + Sy + z = 5

d '" 12 + 5 (- 2) + 3:'" 51.

.)1 + 25+ 1

~ d=I~I=~

46. Since (it x h(~ x b= 1. it it, i

1

=litl2 - ar 'Similarly, (it)( j)2 = I "it 12 - ai

and(~ x ki =1112 - a~

_'"2 _, "2 _, "2 (a xi) + (a xj) + (a xk)

= 31 "112 - (ar + a~ + a~) "" 31112 _1~12

=211f =21'2

Alternate Solution

Let

_, . . .'

a = ali 'I- azJ + Q3k

1~12 = ar + ~ + ai

and it xi =; (ali + azJ + Q3k) x (I) = -a2k + Q3J

_, -2 2 2 (a x i) = a2 + UJ

Similarly. (1' )( )2 = a~ + a~

and C1xki =af + a~ .. (1)( ii + (1 x])2 ~ (1 xki

se (~ + ~ + a~ + at + a{+ a~) = 2 (at + a~ + ail

_, '2

= 2(a)

47. .,' a, b, c are in HP,

2 1 1 'b=a+c

48.

!_3+!'=o abc

So straight line ~ + i + ~ = 0 always passes through a fixed point (1, - 2).

Key Idea :' If A(x). y,), B(x2,YZ) and C(x3, Y3) are the vertices of a triangle, then the co-ordinates of the centroid will be

(Xl + x2 + x3 Yl + Y2 + Y3) 3 ' 3 .

Let D and E are mid points of AB and AC. So co-ordinates of Band Care (- 3, 3) and (5, 3) respectively.

E (3.2)

C(5,3)

Centroid of triangle

== (Xl + Xl + x3 Y! + Y2 + Y3)

3 ' 3

== (1 - 3 + 5 1 + 3 + 3)

3 ' 3

49. Key Idea: If 81 = 0 and 82 = 0 are two circles, then the equation of line passes through the points of intersecrion of SI = 0 and 82 = Q is

SI - S2 = o.

Equation of circles are

81 '" x2 + y2 + 2ax + cy + a == 0

and S2 '" x2 + y2 - 3ax + qy -1 = 0 respectively.

Chord through intersection points P and Q of the circles ~ = 0 and S2 = 0 is SI - 82 '" 0 .

. . ~ + (c - d)y + a + 1 := 0

On comparingit with 5x + by - a = 0, we get

Sa c-d a+l .

S=-b-=-=-U

=> a( - a) = a + 1 =:> a2 + a + 1 == 0 Which gives no real value of a.

.'. The line will passes through P and Q for no value of a.

50. Since circle touches the x-axis and also touches circle with the centre at (0, 3) and radius 2, then

y

C1Cz"'lj+rl

h2 + (k - 3)2 ; (I kl + 2i

h2 + k2 + 9 - 6k '" k2 + 4 + 4! kl :. Locus of centre of circle is x2=-S+6y+4IYI

x2 = lOy ~ 5 t: y > 0)

This equation represents a parabola. Thus locus of the centre of the circle is a parabola.

51. Key Idea: Two circles

x2 + y2 + 2g1x + 2/1Y + CJ = 0

and X2 + y2 + 2g2X + 2IlY + c2 = 0 cuts

orthogonally, then

2,g"lg2 + 21d2 = c1 + '2'

Let the circle is X2 + y2 .~ 4~X + 2/y + C = 0, It cuts circle x'l + y2 =.p2 orthogonally.

So, c;: p2 and it passes through (a. b).

a2 + b2 + 280 + 2/ b + p2 = 0 So, locus of (- g, - I) is

a2 + b2 _ 2ax - 2by + p2 = 0

;;:) 2ax + 2by - err + b2 + p2) = 0

Alternate Solution

Let the centre of required circle is (-g, - f). This circle cut the circle Xl + y2 = p2

orthogonally. The centre and radius of circle Xl + y2 := p2 are (0,0) and p respectively. '

.. g2 + fl. '" p2 + (0 + g)2 + (b + j}2

;;:) g2 + /2 = p2 + 02 + g2 + 2ag + b2

+ 12 + 2bl

=> p2 + a? + b2 + 2ag + 2bf '" 0

Thus the locus of centre is

,Zax + 2by - (02 + b2 + p2)=0

52. LFBF I =- 900~ LOBF' = 450 y

B(O.b)

A{a.O) x

(lnd

ae= b

2 b2

e ;:1--

a2

2, aZez

e =1--- 02-

e2=1_e2

2e2 =1

1 e=Tz

Altem.ate Solution

:. F and F; are foci of an ellipse, whose co-ordinates arc (ae, O)and (-ae, ,0) respectively and co-ordinates of Bare (0, b).

b :. Slope of BF =-ae

" b

, and slope of BF' '" -

ae

LFBF'= 900

- .£. . (.£.) "'-1

ae ae

b2 = a2e2

b2 <te2

c= 1--=1--

a2 rt

2 1

2e =1~e::l..fi

=>

53. Key Idea: A line y '" mx + c is tangent to x2 y2

hyperbola 2'" - ""'2 '" 1, if c2 '" a2m2 - b2.

a b

Line y =- C1,X + i3 is tangent, ifp2 = aZa. 2 - bl, So locus of (a, ~) is y2 = 02x2 - b2

=> a2x2_y2_b2=O

Since this equation represents a hyperbola, so locus or a point P(a, P) is a hyperbola.

54. Since the angle between the line and plane is same as angle between the line and normal to the plane

sin 0= ci + 2] + lk}(zi - j + -5.G.) =!

3../4 + 1 + ').. 3

;;:)., 2 - 2": 'z.fi. '" "J S'+ A

=> 41. = 5+ A 58.
=> 31.= 5
=> A=~
3 55.

Note: The, an~le between a line

x - Xl Y - Yl Z - 1 . and I

-a;-"'~"'~ pane

a2x + bzy + c2z + d2 '" 0 is given by

, il qa2 + ~~ + C1C2 I

smG", ~~2 +bl + cf ~a~ + Iii + c1

The given equations can be rewritten as ~ = J:: ",.E.. and ~ = L/= __!_

3 2 -6 2 -12 -3

.. Angle between the lines is given by

cos a « 6-24+18

..}9 + 4+ 36 ..}4+ 144+ 9

o

..[49..}157 = 0 => 9",,'90°

Note: If q, 11,0; and a2, bz ,c2 are d.r's. of two lines then the angled between them is given by d1a2 + I1bz + cP2

cos e '" -;=~~~~--i==~~= ...

~af + ~2 + cf ~a$. + b§ + c~

Given that

-- 1 1, - 1

P(Av B) "'"6' peA roB) = 4' peA) = 4

-- 1 P(AVB)="6

1

1- peA vB)", "6

1

=> I-P(A)-P(B)+P(AnB)='6

- 1 1

peA) - PCB) + 4'" '6

PCB) =.!. + .!. - .!.

, 4 4 6

1 3

=:} PCB) ="3 and peA) = "4

Clearly PCA nB) =P(A)P(B)

so the events A and B are independent events but not equally likely.

Note: Two events A and B are said to equally likely, if peA) '" P(B) a nd known as independent events, if peA n 8) = peA) P(8).

57. Person 1 has three options to apply.

Similarly, person 2 has three options to apply and person 3 has three options to apply.

Total cases = 33

Now, favourable cases = 3 (An either all has applied for house 1 or 2 or 3)

~ 3 1

So probability = 33 '" 9'

56.

Key Idea: In a poisson distribution, e-).}[

P(X = r) '" -1- (A = mean) . r.

.. P(X=r>1.5)=P(2)+P(3)+ ... ct:) = 1 - «P(O) + P(1))

= 1- (e-2 + e-21x 2) = 1- e~

59. Key Idea : If u is initial velocity of any particle and a is a acceleration, then

v = u + at.

where v is a speed of particle after t seconds.

... (i) ... (ii) ... (iii)

... (iv)

We have VA'" fCt + m) VB = f't

Now, V A = VB ~ f 't '" f(t + m)

S + n = ~ fCt + mi

S""!.f't2

2

n ""!. f(t + ml-!' f' t2

2 2

n=!.fCf'tl -!'f't2

2 f 2 - 2

2nf = {(f ,)2 - f f') t').

z 2nf

t = f'ct'-f)

... (v) ... (Vi)

... (vii)

From Eq. Ciii)

_ fin t-(f'-f)

So from Eqs. (vii) and (viii)

f 2m2 2nf

(f'-fl !,(f'-f) => !' fin2 = 2nU ' - f )

60. Let lizard catch the insect C

and distance covered by insect = S

. k b . S

tune ta en y insect, t = 20

... (viii)

... (i)

Distance covered by lizard = 21 + S

21 + S "" ~ (2}t2 ••• (ii)

=> 21 + 20t '" t2 (using Eq. (i))

=> t2-2Ot-21=0

=> t2 - 21t + t - 21 = 0

=> t(t-21)+I(t-21)=0

=> (t + l)(t - 21) "" 0

=> t "" 21 s.

61. Key Idea: If R is a resultant of two forces P and Q acting on a particle, then

P _ R sin!3 _ _ d Q '- R sin a . - sin(a + 13) an - - sinea + 13) .

Given that R '= ~

p = . R sin e and '" R sin 90°

sin (900 + 9) Q sin (900 + 0)

- Q

Q -:reosB

Q

62.

p

1,. r.:I" 2../2

==> cos 0='3 ==>smO=V1-'9=T

p '" 3R sin 0 = 3R . 2-12

1 3

P '" 2..f2R

~ = 2~ R '" 212 : 3

~:; 3: 2.[2

-)- " , ,

Key Idea : If a ",ali + 02j + 03k,

~ , " i'_ ~ , ,

ti = b1i + b2j + baR and c "" 'Ii + '2j + C;Jk

-+- -=c> -~ 1 al ~ 031

(a Jj c]::= ht b2 bs

'I '2 '3 ~-=c>~ 11 0 -1 1 [abcJ=lx 1 I-x

y x l+x-y

Applying C3 -+ CJ + C1 -

)1 0 0'

=Ix 1 1 I

IY X 1+x

=!; l;xl

= 1 (1 + x) - x = 1

Thus l1 b 7] does not depend upon neither x nor y.

63. Key Idea: If three vectors 1., 'jj and C! are . ~ -'+-'Jo

coplanar, then [ a Jj c J ::= O.

The given points lies in a plane, if

!! ~ I!=o eel -s c, -c»

~ -l[ ~ .11=0

~ -lj~ bl=o

=> (;2 _ ab = 0, Therefore c is GM of a and b.

64.

b2;.CZ b3;C31

b2 b3

~ ~ ~ I~ ~ ~ }..4 ~ b2 b3 '" - ht b2 b3

CJ_cZC3 qCZC3

).4 =-1

So, no real value of J.,. exists.

Alternate Solution

tx (1 + b) A2Ii }.. c.t] =(1 "H' + 1-';]

-+- -=c> 2 -.+ ---)0 ~ ~ -+- -.-'I>

=> A(a + til .(). 1) xl. c)", a.((1) + c)x 1))

4 -+- -.+ -.t -+- ~ -+- -,+

==> A (a + 1)),(ti x c)= a·(c x til

=> A,4{1.(6 x1)}=-ft(it x~)}

-+- -.-'1>---)0

(': [a b c J ¢ 0)

65.

;:;> No real value of A. exists.

Let the resultant of A and B is displaced by x. Then change in net momentum of A and B ,., applied momentum

.. Ax+Bx=H

H x"'A+B

66. We know that

x . Xl x3 X4 .

e e:1+X+2T+3f+41+'"

~ x2 x3 X4

e =l-x+2!-~+4!-""

(( + e~ x2 X4 xc.

=> 2 == 1 + 2T + 41+ 6T + ....

Put

1

x""-

2

e1l2 + e-·l/2 == + (!)2 1... (!)4 l_

2 1 2 21+ 2 4'+····

~ e2:;I=1+(~r ~!+ar ~!+ ....

67. If a> 0, i '" 1, 2, 3,. ... , n Which are nor identical, then

AM of mth power> mth power of AM, if m > 1

::::> x; -t" x~: ... +~>(Xl + x2 ~ ... +.x"y

n 2 i:t Xi ---> n

[II ]2

r X·

i= ~ ~

4~O > (BnOY

n2 802

~>-

n 400

68.

=) n>16

Given Lt .L ~

ti is making an angle 60° with horizontal (upwards).

Therefore ~ should be at 300 with horizontal (downward) as shown in fig.

,)90"

""

u -'-'"

=

v cos 30° = u cos 60"

·I3v 1 2=u'2

u

v == .J3

=

69. Key Idea : If borh roots ofax2 + bx + c = 0 are less ('han 0., then b2 - 4ac ;?; 0, Jell.') > 0 and -b

2a <0..

Let fex) = Xl - 2kx + k2 + k - 5 y

D = 4k2 ~ 4(e + k - 5) = - 4k + 20 ~ 0

k:,:,S

x'

v

... (i)

Also

b --<S 2a

... (ii) ... (iii) ... (iv)

and J(5»O From (ii), k < 5

From (iii), 2S-10k + k2 + k - 5> 0

=) k2-9k+20>O

k2 - Sk - 4k + 20 > 0 (k - 5) (k - 4) > 0

k < 4 and k > 5

... (v)

=) From Ci), (iii) and (iv)

:::;;. k<4

70.

GP.then are in

Since" at, a2 , ... , a",... are in log an> log a,,+ I' log an+ 2' ... , log all + 8 AP.

So, t. =1 .: 3d a"++:d ~: ~ 1

a + 6d a + 7 d a + Sd

. C --'loC -c

Applying 2. :2 ],

C3 -4 C3 -Cl

I a d 2d1

D. =.' a+ 3d d 2d2d =0

I a + 6d d:

(Since two columns are similar)

71. Given f(x - y) '" lex) l(Y) - fCa - x) fCa + y)

Let

x=O=y

fCO) '" (f(o)i - (JCa)i 1 = 1 - (fCa))2 f(a) = 0

fe2a- x)=: f{a - ex - an == fea) fex - a) - fCa - a) j(x) = 0 - lex) ·1

= - lex)

72. Let J(x)="a"x"+a"_lx"-I+ ... +a1x=O,

:::> '

?,3.

al #0

It have roots x = 0 and a > 0 f'(x)=na,.x,,-l + (n-l)an_t x,,-2

+".+a1=0 So definitely its derivative is zero between 0 and a.

So, f 'ex) has a positive root smaller than 0. .

l.et I"" In. cos2 X dx, a> 0 ... (i)

-"1+«'

1= fK

-~

... (ii)

On adding Eqs. (i) and (ii)

=."> 21 "" r .,L ~l + if) cos:! x dx

(1 + aX)

21 = r l' COS2 X dx

21 = I, (COS ~ + 1) dx

~~[ein22x + x )Ix

1

21 =: "2 (n -I- 11:)

1= !!.

2

74. Since the centre of sphere

x2 + y2 + Z2 _ X + Z - 2= 0 is (~. 0, -~) and

radius of sphere

= 1~_!_--I--_!_-+-2 = .Jlo

'1/4 .4 2

Distance of plane from centre of sphere

!+.!-4

- 2 2 = ~6

- .Jl + 4 + 1 vO

So radius of circle "" ~1~ - ~

= ~ 30;218 = M = 1

75. Equation of pair of lines

ax2 + 2 (a + b) xy + by2 = 0

Since

Angle between lines is given by

~ ~

2~H2 -AB tan 9= IA+ BI

1 '" 2~(a+ b)2 - ab la+ bl

a!- + b2 + 2 ab = 4(a2 + b2 + ab) 30:2 + 3b2 + 2ab·=: 0

is

You might also like